Sie sind auf Seite 1von 44

CONSTITUTIONAL LAW

CHAPTER IV THE SEARCH AND SEIZURE PROVISION


THE ANTI-TERRORISM LAW (RA 9372)
NOTE: Applicable provisions of the Human Security Act/Anti-Terrorism Law, Republic Act No. 9372, Approved on
March 6, 2007 and effective on July 15, 2007 (This Law shall be automatically suspended one (1) month before
and two (2) months after the holding of any election)
Sec.18. Period of detention without judicial warrant of arrest.- The provisions of Article 125 of the
Revised Penal Code, notwithstanding, any police or law enforcement personnel, who, having
been duly authorized in writing by the Anti-Terrorism Council has taken custody of a person
charged with or suspected of the crime of terrorism or the crime of conspiracy to commit
terrorism shall, WITHOUT INCURRING ANY CRIMINAL LIABILITY FOR DELAY IN THE
DELIVERY OF DETAINED PERSONS TO THE PROPER JUDICIAL AUTHORITIES, DELIVER
SAID CHARGED OR SUSPECTED PERSON TO THE PROPER JUDICIAL AUTHORITY
WITHIN A PERIOD OF THREE (3) DAYS counted from the moment said charged or suspected
person has been apprehended or arrested, detained, and taken into custody by the said police,
or law enforcement personnel: Provided, That the arrest of those suspected of the crime of
terrorism or conspiracy to commit terrorism must result from the surveillance under Section 7 and
examination of bank deposits under Section 27 of this Act.
The police or law enforcement personnel concerned shall, before detaining the person suspected
of the crime of terrorism, present him or her before any judge at the latters residence or office
nearest the place where the arrest took place at any time of the day or night. It shall be the duty
of the judge, among other things, to ascertain the identity of the police or law enforcement
personnel and the person or persons they have arrested and presented before him or her, to
inquire of them the reasons why they have arrested the person and determine by questioning and
personal observation whether or not the subject has been subjected to any physical, moral or
psychological torture by whom and why. The judge shall then submit a written report of what
he/she had observed when the subject was brought before him to the proper court that has
jurisdiction over the case of the person thus arrested.
The judge shall forthwith submit his report within 3 calendar days from the time the suspect was
brought to his/her residence or office.
Immediately after taking custody of a person charged with or suspected of the crime of terrorism
or conspiracy to commit terrorism, the police or law enforcement personnel shall notify in writing
the judge of the court nearest the place of apprehension or arrest; provided, That where the
arrest is made during Saturdays, Sundays, holidays or after office hours, the written notice shall
be served at the residence of the judge nearest the place where the accused was arrested. The
penalty of 10 years and 1 day to 12 years imprisonment shall be imposed upon the police or law
enforcement personnel who fail to notify any judge as provided in the preceding paragraph.
Sec.19. Period of Detention in the event of an actual or imminent terrorist attack.- In the vent of
an actual or imminent terrorist attack,, suspects may not be detained for more than three days
without the written approval of a municipal, city, provincial or regional official of a Human Rights
Commission, or judge of the municipal, regional trial court, the Sandiganbayan or a justice of the
Court of Appeals nearest the place of arrest. If the arrest is made during Saturdays, Sundays or
holidays, or after office hours, the arresting police of law enforcement personnel shall bring the
person thus arrested to the residence of any of the officials mentioned above that is nearest the
place where the accused was arrested. The approval in writing of any of the said officials shall be
secured by the police or law enforcement personnel concerned within five days after the date of
the detention of the persons concerned; Provided, however, That within three days after the
detention the suspects whose connection with the terror attack or threat is not established, shall
be released immediately.
Sec.26. provides that persons who have been charged with terrorism or conspiracy to commit
terrorismeven if they have been granted bail because evidence of guilt is not strongcan be:
1. Detained under house arrest;

2. Restricted from traveling; and/or


3. Prohibited from using any cellular phones, computers, or other means of communications
with people outside their residence.
Sec.39. Seizure and Sequestration.- The deposits and their outstanding balances, placements,
trust accounts, assets, and records in any bank or financial institution, moneys, businesses,
transportation and communication equipment, supplies and other implements, and property of
whatever kind and nature belonging:
To any person charged with or suspected of the crime of terrorism or conspiracy to commit
terrorism;
to a judicially declared and outlawed terrorist organization or group of persons;
to a member of such judicially declared and outlawed organization, association or group of
persons,
-shall be seized, sequestered, and frozen in order to prevent their use, transfer or conveyance for
purposes that are inimical to the safety and security of the people or injurious to the interest of the
State.
The accused or suspect may withdraw such sums as are reasonably needed by his family
including the services of his counsel and his familys medical needs upon approval of the court.
He or she may also use any of his property that is under seizure or sequestration or frozen
because of his/her indictment as a terrorist upon permission of the court for any legitimate
reason.
Sec.40. The seized, sequestered and frozen bank depositsshall be deemed property held in
trust by the bank or financial institution and that their use or disposition while the case is pending
shall be subject to the approval of the court before which the case or cases are pending.
Sec.41. If the person suspected as terrorist is acquitted after arraignment or his case dismissed
before his arraignment by a competent court, the seizureshall be lifted by the investigating body
or the competent court and restored to him without delay. The filing of an appeal or motion for
reconsideration shall not stay the release of said funds from seizure, sequestration and freezing.
If convicted, said seized, sequestered and frozen assets shall automatically forfeited in favor of
the government.

CASES
UY VS BUREAU OF INTERNAL REVENUE, 344 SCRA 36
Search and Seizure Requisites of a Valid Search Warrant
In Sept 1993, Rodrigo Abos, a former employee of UPC reported to the BIR that Uy Chin Ho aka Frank Uy,
manager of UPC, was selling thousands of cartons of canned cartons without issuing a report. This is a violation of
Sec 253 & 263 of the Internal Revenue Code. In Oct 1993, the BIR requested before RTC Cebu to issue a search
warrant. Judge Gozo-Dadole issued a warrant on the same day. A second warrant was issued which contains the
same substance but has only one page, the same was dated Oct 1st 2003. These warrants were issued for the
alleged violation by Uy of Sec 253. A third warrant was issued on the same day for the alleged violation of Uy of
Sec 238 in relation to sec 263. On the strength of these warrants, agents of the BIR, accompanied by members of
the PNP, on 2 Oct 1993, searched the premises of the UPC. They seized, among other things, the records and
documents of UPC. A return of said search was duly made by Labaria with the RTC of Cebu. UPC filed a motion to
quash the warrants which was denied by the RTC. They appealed before the CA via certiorari. The CA dismissed
the appeal for a certiorari is not the proper remedy.
ISSUE: Whether or not there was a valid search warrant issued.
HELD: The SC ruled in favor of UPC and Uy in a way for it ordered the return of the seized items but sustained the
validity of the warrant. The SC ruled that the search warrant issued has not met some basic requisites of validity. A
search warrant must conform strictly to the requirements of the foregoing constitutional and statutory provisions.
These requirements, in outline form, are:
(1) the warrant must be issued upon probable cause;
(2) the probable cause must be determined by the judge himself and not by the applicant or any other person;

(3) in the determination of probable cause, the judge must examine, under oath or affirmation, the complainant
and such witnesses as the latter may produce; and
(4) the warrant issued must particularly describe the place to be searched and persons or things to be seized.
The SC noted that there has been inconsistencies in the description of the place to be searched as indicated in the
said warrants. Also the thing to be seized was not clearly defined by the judge. He used generic itineraries. The
warrants were also inconsistent as to who should be searched. One warrant was directed only against Uy and the
other was against Uy and UPC. The SC however noted that the inconsistencies wered cured by the issuance of the
latter warrant as it has revoked the two others.
Section 2, Article III of the Constitution guarantees the right of the people against unreasonable searches and
seizures:
The right of the people to be secure in their persons, houses, papers, and effects against unreasonable searches
and seizures of whatever nature and for any purpose shall be inviolable, and no search warrant or warrant of arrest
shall issue except upon probable cause to be determined personally by the judge after examination under oath or
affirmation of the complainant and the witnesses he may produce, and particularly describing the place to be
searched and the persons or things to be seized.
NOTES:
Rule 126 of the Rules of Court provides:
SEC. 3. Requisite for issuing search warrant. A search warrant shall not issue but upon probable cause
in connection with one specific offense to be determined personally by the judge after examination under
oath or affirmation of the complainant and the witnesses he may produce, and particularly describing the
place to be searched and the things to be seized.
SEC. 4. Examination of complainant; record. The judge must, before issuing the warrant, personally
examine in the form of searching questions and answers, in writing and under oath the complainant and
any witnesses he may produce on facts personally known to them and attach to the record their sworn
statements together with any affidavits submitted.

PEOPLE VS COURT OF APPEALS, 291 SCRA 400


The general rule is that search warrants must be served during the daytime (protect the public from the
abrasiveness of official intrusions). Exception: a search at any reasonable hour of day or night may be made when
the application asserts that the property in on the person or place ordered to be searched. Absence of abuse of
discretion, a search conducted at night where so allowed is not improper
Abigails Variety Store VOID warrant the claim that the place actually searched although not the one specified
in the warrant is exactly what they had in view when they applied for the warrant is unacceptable. What is
material in determining the validity of the warrant is the place stated in the warrant, not the one they had in their
thoughts; particularization of description may properly be done only by the judge and only in the warrant itself.
A petition for certiorari has been filed to invalidate the order of Judge Casanova which quashed search warrant
issued by Judge Bacalla and declared inadmissible for any purpose the items seized under the warrant.
An application for a search warrant was made by S/Insp Brillantes against Mr. Azfar Hussain who had allegedly in
his possession firearms and explosives at Abigail Variety Store, Apt 1207 Area F. Bagon Buhay Avenue, Sarang
Palay, San Jose Del Monte, Bulacan. The following day Search Warrant No. 1068 was issued but was served not
at Abigail Variety Store but at Apt. No. 1, immediately adjacent to Abigail Variety Store resulting in the arrest of 4
Pakistani nationals and the seizure of a number of different explosives and firearms.
ISSUE: WON a search warrant was validly issued as regard the apartment in which private respondents were then
actually residing, or more explicitly, WON that particular apartment had been specifically described in the warrant.
HELD: The ambiguity lies outside the instrument, arising from the absence of a meeting of minds as to the place to
be searched between the applicants for the warrant and the Judge issuing the same; and what was done was to
substitute for the place that the Judge had written down in the warrant, the premises that the executing officers had
in their mind. This should not have been done. It is neither fair nor licit to allow police officers to search a place
different from that stated in the warrant on the claim that the place actually searched although not that specified
in the warrant is exactly what they had in view when they applied for the warrant and had demarcated in their
supporting evidence. What is material in determining the validity of a search is the place stated in the warrant itself,
not what the applicants had in their thoughts, or had represented in the proofs they submitted to the court issuing
the warrant.
The place to be searched, as set out in the warrant, cannot be amplified or modified by the officers' own personal
knowledge of the premises, or the evidence they adduced in support of their application for the warrant. Such a
change is proscribed by the Constitution which requires inter alia the search warrant to particularly describe the
place to be searched as well as the persons or things to be seized. It would concede to police officers the power of

choosing the place to be searched, even if it not be that delineated in the warrant. It would open wide the door to
abuse of the search process, and grant to officers executing a search warrant that discretion which the Constitution
has precisely removed from them. The particularization of the description of the place to be searched may properly
be done only by the Judge, and only in the warrant itself; it cannot be left to the discretion of the police officers
conducting the search.

PEOPLE VS ARUTA, 288 SCRA 626


Search and Seizure Informers Tip
As a general rule: Validity of warrantless searches and seizures as a result of an informers tip
On December 13, 1988, Olongapo Police received a tip regarding 8.5 kilos of marijuana
In the morning of 13 Dec 1988, the law enforcement officers received information from an informant named Benjie
that a certain Aling Rosa would be leaving for Baguio City on 14 Dec 1988 and would be back in the afternoon of
the same day carrying with her a large volume of marijuana; At 6:30 in the evening of 14 Dec 1988, Aruta alighted
from a Victory Liner Bus carrying a travelling bag even as the informant pointed her out to the law enforcement
officers; NARCOM officers approached her and introduced themselves as NARCOM agents; When asked by Lt.
Abello about the contents of her travelling bag, she gave the same to him; When they opened the same, they
found dried marijuana leaves; Aruta was then brought to the NARCOM office for investigation.
ISSUE: Whether or not the conducted search and seizure is constitutional.
HELD: The SC ruled in favor of Aruta and has noted that some drug traffickers are being freed due to technicalities.
Aruta cannot be said to be committing a crime. Neither was she about to commit one nor had she just committed a
crime. Aruta was merely crossing the street and was not acting in any manner that would engender a reasonable
ground for the NARCOM agents to suspect and conclude that she was committing a crime. It was only when the
informant pointed to Aruta and identified her to the agents as the carrier of the marijuana that she was singled out
as the suspect. The NARCOM agents would not have apprehended Aruta were it not for the furtive finger of the
informant because, as clearly illustrated by the evidence on record, there was no reason whatsoever for them to
suspect that accused-appellant was committing a crime, except for the pointing finger of the informant. The SC
could neither sanction nor tolerate as it is a clear violation of the constitutional guarantee against unreasonable
search and seizure. Neither was there any semblance of any compliance with the rigid requirements of probable
cause and warrantless arrests. Consequently, there was no legal basis for the NARCOM agents to effect a
warrantless search of Arutas bag, there being no probable cause and the accused-appellant not having been
lawfully arrested. Stated otherwise, the arrest being incipiently illegal, it logically follows that the subsequent search
was similarly illegal, it being not incidental to a lawful arrest. The constitutional guarantee against unreasonable
search and seizure must perforce operate in favor of accused-appellant. As such, the articles seized could not be
used as evidence against accused-appellant for these are fruits of a poisoned tree and, therefore, must be
rejected, pursuant to Article III, Sec. 3(2) of the Constitution.
NOTES:
When is a warrantless search allowed?
1. Warrantless search incidental to a lawful arrest recognized under Section 12, Rule 126 of the Rules
of Court 8 and by prevailing jurisprudence;
2. Seizure of evidence in plain view, the elements of which are:
a) a prior valid intrusion based on the valid warrantless arrest in which the police are legally present
in the pursuit of their official duties;
b) the evidence was inadvertently discovered by the police who had the right to be where they are;
c) the evidence must be immediately apparent, and
d) plain view justified mere seizure of evidence without further search;
3. Search of a moving vehicle. Highly regulated by the government, the vehicles inherent mobility
reduces expectation of privacy especially when its transit in public thoroughfares furnishes a highly
reasonable suspicion amounting to probable cause that the occupant committed a criminal activity;
4. Consented warrantless search;
5. Customs search;
6. Stop and Frisk; and
7. Exigent and Emergency Circumstances.

PEOPLE VS MONTILLA, 284 SCRA 703


Political Law Search and Seizure Informers Tip Warrantless Arrest
Dasmarinas, Cavite Police received a tip on June 19, 1994. The marijuana courier will alight at the waiting shed of
Barangay Salitran, Dasmarinas, Cavite, 28 kilos of marijuana.

On 19 June 1994 at about 2pm, police officers Talingting and Clarin were informed by an asset that a drug courier
would be arriving from Baguio to Dasmarias carrying an undetermined amount of marijuana. The next day, the
informant pointed at Montilla as the courier who was waiting in a waiting shed Brgy Salitran, Dasmarias. Montilla
was then apprehended and he was caught in possession of a bag and a carton worth 28 kilos of marijuana. Montilla
denied the allegation and he said he came to Cavite from Baguio for work and he does not have any effects with
him at that time except for some pocket money. He was sentenced to death thereafter. He averred that the search
and seizure conducted was illegal for there was no warrant and that he should have been given the opportunity to
cross examine the informant. He said that if the informant has given the cops the information about his arrival as
early as the day before his apprehension, the cops should have ample time to secure a search warrant.
ISSUE: Whether or not the warrantless arrest conducted is legal.
HELD: The SC ruled that the warrantless arrest is legal and so was the warrantless search. Sec 2 Art 3 of the
Constitution has its exception when it comes to warrantless searches, they are:
(1) customs searches;
(2) searches of moving vehicles,
(3) seizure of evidence in plain view;
(4) consented searches;
(5) searches incidental to a lawful arrest;
(6) stop and frisk measures have been invariably recognized as the traditional exceptions.
In the case at bar, it should be noted that the information relayed by informant to the cops was that there would be
delivery of marijuana at Barangay Salitran by a courier coming from Baguio in the early morning of June 20, 1994.
Even assuming that the policemen were not pressed for time, this would be beside the point for, under these
circumstances; the information relayed was too sketchy and not detailed enough for the obtention of the
corresponding arrest or search warrant. While there is an indication that the informant knew the courier, the records
do not reveal that he knew him by name.
On such bare information, the police authorities could not have properly applied for a warrant, assuming that they
could readily have access to a judge or a court that was still open by the time they could make preparations for
applying therefor, and on which there is no evidence presented by the defense. In determining the opportunity for
obtaining warrants, not only the intervening time is controlling but all the coincident and ambient circumstances
should be considered, especially in rural areas.
A legitimate warrantless arrest, as above contemplated, necessarily cloaks the arresting police officer with authority
to validly search and seize from the offender
(1) dangerous weapons, and
(2) those that may be used as proof of the commission of an offense.

PEOPLE VS RACHO, GR186529


On May 19, 2003, a confidential agent of the police transacted through cellular phone with appellant for the
purchase of shabu. The agent reported the transaction to the police authorities who immediately formed a team to
apprehend the appellant. The team members posted themselves along the national highway in Baler, Aurora, and
at around 3:00 p.m. of the same day, a Genesis bus arrived in Baler. When appellant alighted from the bus, the
confidential agent pointed to him as the person he transacted with, and when the latter was about to board a
tricycle, the team approached him and invited him to the police station as he was suspected of carrying shabu.
When he pulled out his hands from his pants pocket, a white envelope slipped therefrom which, when opened,
yielded a small sachet containing the suspected drug. The team then brought appellant to the police station for
investigation and the confiscated specimen was marked in the presence of appellant. The field test and laboratory
examinations on the contents of the confiscated sachet yielded positive results for methamphetamine
hydrochloride. Appellant was charged in two separate informations, one for violation of Section 5 of R.A. 9165, for
transporting or delivering; and the second, of Section 11 of the same law for possessing, dangerous drugs. During
the arraignment, appellant pleaded "Not Guilty" to both charges.
On July 8, 2004, the RTC rendered a Joint Judgment convicting appellant of Violation of Section 5, Article II, R.A.
9165 but acquitted him of the charge of Violation of Section 11, Article II, R.A. 9165. On appeal, the CA affirmed the
RTC decision. The appellant brought the case to SC assailing for the first time he legality of his arrest and the
validity of the subsequent warrantless search.
ISSUE: Whether or not the appellant has a ground to assail the validity of his arrest.
HELD: The long standing rule in this jurisdiction is that "reliable information" alone is not sufficient to justify a
warrantless arrest. The rule requires, in addition, that the accused perform some overt act that would indicate that
he has committed, is actually committing, or is attempting to commit an offense. We find no cogent reason to depart
from this well-established doctrine. Appellant herein was not committing a crime in the presence of the police
officers. Neither did the arresting officers have personal knowledge of facts indicating that the person to be arrested
had committed, was committing, or about to commit an offense. At the time of the arrest, appellant had just alighted

from the Gemini bus and was waiting for a tricycle. Appellant was not acting in any suspicious manner that would
engender a reasonable ground for the police officers to suspect and conclude that he was committing or intending
to commit a crime. Were it not for the information given by the informant, appellant would not have been
apprehended and no search would have been made, and consequently, the sachet of shabu would not have been
confiscated. Neither was the arresting officers impelled by any urgency that would allow them to do away with the
requisite warrant. As testified to by Police Officer 1 Aurelio Iniwan, a member of the arresting team, their office
received the "tipped information" on May 19, 2003. They likewise learned from the informant not only the
appellants physical description but also his name. Although it was not certain that appellant would arrive on the
same day (May 19), there was an assurance that he would be there the following day(May 20). Clearly, the police
had ample opportunity to apply for a warrant.

PEOPLE VS CLAUDIO, 160 SCRA 646


There is probable cause in the Warrantless search of a bag (behind him in a bus) by a NARCOM agent when he
allegedly smelled marijuana therein. By reason of his training, he could smell marijuana and therefore, he has
personal knowledge and therefore, probable cause was present making the search legal.

PEOPLE VS AMINNUDIN Y AHNI, JULY 6, 1988


M/V Wilcon; marijuana not caught in flagrante delicto; search was unreasonable; evidence inadmissible.
Tip received by the Iloilo City Police from informant in Zamboanga city that accused has marijuana in his bag does
not amount to probable cause because the Iloilo Police have no personal knowledge. Further, there was plenty of
time to secure a search warrant from the court.

PEOPLE VS MALMSTEDT, 198 SCRA 401


In an information filed against the accused- appellant Mikael Malmstead was charged before the RTC of La
Trinidad, Benguet, for violation of Section 4, Art. II of Republic Act 6425, as amended, otherwise known as the
Dangerous Drugs Act of 1972, as amended.
Accused Mikael Malmstedt, a Swedish national, entered the Philippines for the third time in December 1988 as a
tourist. He had visited the country sometime in 1982 and 1985. In the evening of 7 May 1989, accused left for
Baguio City. Upon his arrival thereat in the morning of the following day, he took a bus to Sagada and stayed in that
place for two (2) days. Then in the 7 in the morning of May 11, 1989, the accused went to Nangonogan bus stop in
Sagada. At about 8: 00 o'clock in the morning of that same day (11 May 1989), Captain Alen Vasco, the
Commanding Officer of the First Regional Command (NARCOM) stationed at Camp Dangwa, ordered his men to
set up a temporary checkpoint at Kilometer 14, Acop, Tublay, Mountain Province, for the purpose of checking all
vehicles coming from the Cordillera Region. The order to establish a checkpoint in the said area was prompted by
persistent reports that vehicles coming from Sagada were transporting marijuana and other prohibited drugs.
Moreover, information was received by the Commanding Officer of NARCOM, that same morning that a Caucasian
coming from Sagada had in his possession prohibited drugs. The group composed of seven (7) NARCOM officers,
in coordination with Tublay Police Station, set up a checkpoint at the designated area at about 10:00 o'clock in the
morning and inspected all vehicles coming from the Cordillera Region.
The two (2) NARCOM officers started their inspection from the front going towards the rear of the bus. Accused
who was the sole foreigner riding the bus was seated at the rear thereof.
During the inspection, CIC Galutan noticed a bulge on accused's waist. Suspecting the bulge on accused's waist to
be a gun, the officer asked for accused's passport and other identification papers. When accused failed to comply,
the officer required him to bring out whatever it was that was bulging on his waist. The bulging object turned out to
be a pouch bag and when accused opened the same bag, as ordered, the officer noticed four (4) suspiciouslooking objects wrapped in brown packing tape, prompting the officer to open one of the wrapped objects. The
wrapped objects turned out to contain hashish, a derivative of marijuana.
Thereafter, accused was invited outside the bus for questioning. But before he alighted from the bus, accused
stopped to get two (2) travelling bags from the luggage carrier. Upon stepping out of the bus, the officers got the
bags and opened them. A teddy bear was found in each bag. Feeling the teddy bears, the officer noticed that there
were bulges inside the same which did not feel like foam stuffing. It was only after the officers had opened the bags
that accused finally presented his passport.
Accused was then brought to the headquarters of the NARCOM at Camp Dangwa, La Trinidad, Benguet for further
investigation. At the investigation room, the officers opened the teddy bears and they were found to also contain
hashish. Representative samples were taken from the hashish found among the personal effects of accused and
the same were brought to the PC Crime Laboratory for chemical analysis.

In the chemistry report, it was established that the objects examined were hashish. a prohibited drug which is a
derivative of marijuana. Thus, an information was filed against accused for violation of the Dangerous Drugs Act.
ACCUSEDS DEFENSE: During the arraignment, accused entered a plea of "not guilty." For his defense, he raised
the issue of illegal search of his personal effects. He also claimed that the hashish was planted by the NARCOM
officers in his pouch bag and that the two (2) travelling bags were not owned by him, but were merely entrusted to
him by an Australian couple whom he met in Sagada. He further claimed that the Australian couple intended to take
the same bus with him but because there were no more seats available in said bus, they decided to take the next
ride and asked accused to take charge of the bags, and that they would meet each other at the Dangwa Station.
The trial court found the guilt of the accused Mikael Malmstedt established beyond reasonable doubt.
Seeking the reversal of the decision of the trial court finding him guilty of the crime charged, accused argues that
the search of his personal effects was illegal because it was made without a search warrant and, therefore, the
prohibited drugs which were discovered during the illegal search are not admissible as evidence against him.
ISSUE: Whether or Not the contention of the accused is valid, and therefore the RTC ruling be reversed.
HELD: The Constitution guarantees the right of the people to be secure in their persons, houses, papers and
effects against unreasonable searches and seizures. However, where the search is made pursuant to a lawful
arrest, there is no need to obtain a search warrant. A lawful arrest without a warrant may be made by a peace
officer or a private person under the following circumstances.
Sec. 5 Arrest without warrant; when lawful. A peace officer or a private person may, without a
warrant, arrest a person:
(a) When, in his presence, the person to be arrested has committed is actually committing, or is
attempting to commit an offense;
(b) When an offense has in fact just been committed, and he has personal knowledge of facts
indicating that the person to be arrested has committed it; and
(c) When the person to be arrested is a prisoner who has escaped from a penal establishment or
place where he is serving final judgment or temporarily confined while his case is pending, or has
escaped while being transferred from one confinement to another.
Accused was searched and arrested while transporting prohibited drugs (hashish). A crime was actually being
committed by the accused and he was caught in flagrante delicto. Thus, the search made upon his personal effects
falls squarely under paragraph (1) of the foregoing provisions of law, which allow a warrantless search incident to a
lawful arrest. While it is true that the NARCOM officers were not armed with a search warrant when the search was
made over the personal effects of accused, however, under the circumstances of the case, there was sufficient
probable cause for said officers to believe that accused was then and there committing a crime.
Probable cause has been defined as such facts and circumstances which could lead a reasonable, discreet and
prudent man to believe that an offense has been committed, and that the objects sought in connection with the
offense are in the place sought to be searched. Warrantless search of the personal effects of an accused has been
declared by this Court as valid, because of existence of probable cause, where the smell of marijuana emanated
from a plastic bag owned by the accused, 10 or where the accused was acting suspiciously, 11 and attempted to
flee.
The appealed judgment of conviction by the trial court is hereby affirmed. Costs against the accused-appellant.

STONEHILL VS DIOKNO, JUNE 19, 1967


General warrant for violation of CB Laws, TCC, NIRC and RPC
Search and Seizure General Warrants Abandonment of the Moncado Doctrine
Stonehill et al and the corporation they form were alleged to have committed acts in violation of Central Bank
Laws, Tariff and Customs Laws, Internal Revenue (Code) and Revised Penal Code. By the strength of this
allegation a search warrant was issued against their persons and their corporation. The warrant provides authority
to search the persons above-named and/or the premises of their offices, warehouses and/or residences, and to
seize and take possession of the following personal property to wit:
Books of accounts, financial records, vouchers, correspondence, receipts, ledgers, journals, portfolios, credit
journals, typewriters, and other documents and/or papers showing all business transactions including
disbursements receipts, balance sheets and profit and loss statements and Bobbins (cigarette wrappers).
The documents, papers, and things seized under the alleged authority of the warrants in question may be split into
(2) major groups, namely:
a. those found and seized in the offices of the aforementioned corporations and
b. those found seized in the residences of petitioners herein.
Stonehill averred that the warrant is illegal for:
(1) they do not describe with particularity the documents, books and things to be seized;
(2) cash money, not mentioned in the warrants, were actually seized;

(3) the warrants were issued to fish evidence against the aforementioned petitioners in deportation cases filed
against them;
(4) the searches and seizures were made in an illegal manner; and
(5) the documents, papers and cash money seized were not delivered to the courts that issued the warrants, to
be disposed of in accordance with law.
The prosecution counters, invoking the Moncado doctrine, that the defects of said warrants, if any, were cured by
petitioners consent; and (3) that, in any event, the effects seized are admissible in evidence against them. In short,
the criminal cannot be set free just because the government blunders.
ISSUE: Whether or not the search warrant issued is valid.
HELD: The SC ruled in favor of Stonehill et al. The SC emphasized however that Stonehill et al cannot assail the
validity of the search warrant issued against their corporation for Stonehill are not the proper party hence has no
cause of action. It should be raised by the officers or board members of the corporation. The constitution protects
the peoples right against unreasonable search and seizure. It provides; (1) that no warrant shall issue but upon
probable cause, to be determined by the judge in the manner set forth in said provision; and (2) that the warrant
shall particularly describe the things to be seized. In the case at bar, none of these are met. The warrant was
issued from mere allegation that Stonehill et al committed a violation of Central Bank Laws, Tariff and Customs
Laws, Internal Revenue (Code) and Revised Penal Code. In other words, no specific offense had been alleged in
said applications. The averments thereof with respect to the offense committed were abstract. As a consequence, it
was impossible for the judges who issued the warrants to have found the existence of probable cause, for the same
presupposes the introduction of competent proof that the party against whom it is sought has performed particular
acts, or committed specific omissions, violating a given provision of our criminal laws. As a matter of fact, the
applications involved in this case do not allege any specific acts performed by herein petitioners. It would be a legal
heresy, of the highest order, to convict anybody of a violation of Central Bank Laws, Tariff and Customs Laws,
Internal Revenue (Code) and Revised Penal Code, as alleged in the aforementioned applications without
reference to any determinate provision of said laws or codes.
The grave violation of the Constitution made in the application for the contested search warrants was compounded
by the description therein made of the effects to be searched for and seized, to wit:
Books of accounts, financial records, vouchers, journals, correspondence, receipts, ledgers, portfolios, credit
journals, typewriters, and other documents and/or papers showing all business transactions including disbursement
receipts, balance sheets and related profit and loss statements.
Thus, the warrants authorized the search for and seizure of records pertaining to all business transactions of
Stonehill et al, regardless of whether the transactions were legal or illegal. The warrants sanctioned the seizure of
all records of Stonehill et al and the aforementioned corporations, whatever their nature, thus openly contravening
the explicit command of the Bill of Rights that the things to be seized be particularly described as well as
tending to defeat its major objective: the elimination of general warrants. The Moncado doctrine is likewise
abandoned and the right of the accused against a defective search warrant is emphasized.

BACHE VS RUIZ, 37 SCRA 823


Search and Seizure Personal Examination of the Judge
The clerk of court received the evidence of the applicant for a search warrant
On 24 Feb 1970, Commissioner Vera of Internal Revenue, wrote a letter addressed to J Ruiz requesting the
issuance of a search warrant against petitioners for violation of Sec 46(a) of the NIRC, in relation to all other
pertinent provisions thereof, particularly Sects 53, 72, 73, 208 and 209, and authorizing Revenue Examiner de
Leon make and file the application for search warrant which was attached to the letter. The next day, de Leon and
his witnesses went to CFI Rizal to obtain the search warrant. At that time J Ruiz was hearing a certain case; so, by
means of a note, he instructed his Deputy Clerk of Court to take the depositions of De Leon and Logronio. After the
session had adjourned, J Ruiz was informed that the depositions had already been taken. The stenographer read
to him her stenographic notes; and thereafter, J Ruiz asked respondent Logronio to take the oath and warned him
that if his deposition was found to be false and without legal basis, he could be charged for perjury. J Ruiz signed
de Leons application for search warrant and Logronios deposition. The search was subsequently conducted.
ISSUE: Whether or not there had been a valid search warrant.
HELD: The SC ruled in favor of Bache on three grounds.
1. J Ruiz failed to personally examine the complainant and his witness.
Personal examination by the judge of the complainant and his witnesses is necessary to enable him to
determine the existence or non-existence of a probable cause.
2. The search warrant was issued for more than one specific offense.
The search warrant in question was issued for at least four distinct offenses under the Tax Code. As ruled
in Stonehill Such is the seriousness of the irregularities committed in connection with the disputed search
warrants, that this Court deemed it fit to amend Section 3 of Rule 122 of the former Rules of Court that a

search warrant shall not issue but upon probable cause in connection with one specific offense. Not
satisfied with this qualification, the Court added thereto a paragraph, directing that no search warrant shall
issue for more than one specific offense.
3. The search warrant does not particularly describe the things to be seized.
The documents, papers and effects sought to be seized are described in the Search Warrant
Unregistered and private books of accounts (ledgers, journals, columnars, receipts and disbursements
books, customers ledgers); receipts for payments received; certificates of stocks and securities; contracts,
promissory notes and deeds of sale; telex and coded messages; business communications, accounting
and business records; checks and check stubs; records of bank deposits and withdrawals; and records of
foreign remittances, covering the years 1966 to 1970.
The description does not meet the requirement in Art III, Sec. 1, of the Constitution, and of Sec. 3, Rule 126 of the
Revised Rules of Court, that the warrant should particularly describe the things to be seized.
A search warrant may be said to particularly describe the things to be seized when the description therein is as
specific as the circumstances will ordinarily allow or when the description expresses a conclusion of fact not of law
by which the warrant officer may be guided in making the search and seizure or when the things described are
limited to those which bear direct relation to the offense for which the warrant is being issued.
May a judge deputize his Clerk of Court to take the deposition of the applicant for a Search Warrant subject to
clarificatory questions after his hearing in other cases?
No. As held in Bache vs. Ruiz, 37 SCRA 823, the examination of the complainant ant the witnesses he may
produce must be done personally by the judge. Otherwise, the warrant shall be void. As such, the SC held in
PENDON VS. CA, November 16, 1990 that when the questions asked to the applicant for a search warrant was
pre-typed, the same is not valid since there could have been no searching questions.

SECRETARY OF JUSTICE VS MARCOS, 76 SCRA 301


Illegal possession of firearm and violation of CB Laws
Search and Seizure
On March 31, 1971, Amansec went to Baguio and passed by a house at 47 Ledesma Street, Baguio; he was
attracted by the sight of several persons inside the house; he peeped from outside the house and when the curtain
was moved he saw a Buddha that was inside the house; he observed what was going on inside the house and he
heard someone say that the golden Buddha was actually for sale and when he observed them closer he overheard
that it was being offered for sale for 100,000 pesos by Rogelio Roxas; he saw the Buddha and firearms and some
bullets inside the house. By these facts, Colonel Calano requested for a warrant from J Marcos at about 12
midnight on Apr 4, 1971. Due to the urgency he issued the warrant. And eventually the golden Buddha and some
firearms were seized from Roxass house. Santos assailed the warrant averring that the search warrant was not
limited to one offense covering both illegal possession of firearms and violation of Central Bank rules and
regulations; that it did not particularly describe the property to be seized; that he did not carefully examine under
oath the applicant and his witnesses; that articles not mentioned were taken; and that thereafter the return and the
inventory although appearing to have been prepared on said date were not actually submitted to respondent Judge
until April 13, 1971 and the objects seized delivered only about a week later on April 19.
ISSUE: Whether or not the search warrant issued by Judge Marcos is valid.
HELD: The SC ruled in favor Judge Marcos and had basically affirmed the decision of appellate Judge Gatamaitan.
Taking into consideration to nature of the articles so described, it is clear that no other more adequate and detailed
description could be given, particularly because it is difficult to give a particular description of the contents thereof,
The description so made substantially complies with the legal provisions because the officer of the law who
executed the warrant was thereby placed in a position enabling him to Identify the articles in question, which he
did, so that here, since certainly, no one would be mistaken in Identifying the Buddha, whose image is well
known, and even the firearms and ammunition because these were those without permit to possess, and all located
at 47 Ledesma St., Baguio City, so far as description was concerned, the search warrant perhaps could not be said
to have suffered fatal defects.

CASTRO VS PABALAN, April 30, l976


The search warrant is implemented in an adjoining Barrio of Bangar, La Union
Judge Pabalan ordered the issuance of a search warrant despite failure of the application of Lumang or the warrant
itself to specify the offense, to examine the applicant as well as his witnesses on the part of the Judge, and to
describe with particularity the place to be searched and the things to be seized. Judge never refuted the assertions
when required to answer. Application alleged that applicants wee informed and claimed that they verified the report
that Maria Castro and Co Ling are in possession of narcotics and other contraband in Barrio Padasil, Bangar, La

Union without specifying the particular place in the Barrio. No complete description of the goods and inquiry was
brief. Upon actual search, it turned out that it was in Barrio Ma. Cristina and not in Padasil.
ISSUE: Whether or not the search warrant is validly issued.
HELD: Search warrant issued illegal for violation of the 1935 Constitution and the Rules of Court because the two
basic requirements are not complied with: (a) no warrant shall issue but upon probable cause, (b) the warrant shall
particularly describe the things to be seized, thus, a general warrant. However, things seized cannot be returned
and shall be destroyed, except the liquors, playing cards, distilled water and five bottles of Streptomycin.

ASIAN SURETY VS HERRERA, 52 SCRA 312


Search warrant for estafa, falsification, tax evasion and insurance fraud is a general warrant despite two carloads of
evidence seized
May a Search Warrant be issued for the crimes of Search Warrant for estafa, falsification, tax evasion and
insurance fraud?
No, such would be a general warrant and violates the rule that a warrant shall be issued for one (1) specific
offense. (Asian Surety vs. Herrera, 54 SCRA 312)

COLLECTOR OF CUSTOMS VS VILLALUZ, June 18, 1976

VIDUYA VS BERDIAGO, 73 SCRA 553


Except in the case of the search of a dwelling house, persons exercising police authority under the customs law
may effect search and seizure without a search warrant in the enforcement of customs laws.
Respondent Berdiago is the owner of a Rolls Royce car, Model 1966, which arrived in the Port of Manila on
January 8, 1968. However, the petitioner, Jose Viduya, then Collector of Customs of Manila, obtained reliable
intelligence that fraudulent documents were used by Berdiago in securing the release of the car from the Bureau of
Customs, making it appear therein that the car was a 1961 model instead of a 1966 one, thus enabling respondent
to pay a much lower customs duty.
There was, accordingly, a formal demand for the payment of the sum to cover the deficiency, respondent
manifesting his willingness to do so but failing to live up to his promise. As the car was kept in a dwelling house at
the Yabut Compound, two officials of the Customs Police Service as duly authorized agents of petitioner, applied to
respondent Judge for a warrant to search said dwelling house and to seize the Rolls Royce car found therein.
Berdiago filed a motion to quash the search warrant issued by the court based on lack of probable cause to issue
the warrant. Collector Viduya opposed, alleging that Berdiago could not rely on the constitutional right against
unreasonable search and seizure because it was not shown that he owned the dwelling house which was
searched. Nonetheless, respondent Judge in the challenged order quashed such search warrant.
ISSUE: Whether or not respondent Judge committed grave abuse of discretion in quashing the warrant
HELD: The Court opined that except in the case of the search of a dwelling house, persons exercising police
authority under the customs law may effect search and seizure without a search warrant in the enforcement of
customs laws. There is justification then for the insistence on the part of private respondent that probable cause be
shown. So respondent Judge found in issuing the search warrant.
Apparently, he was persuaded to quash it when he noted that the warrant for seizure and detention came later than
its issuance. In thus acting, respondent Judge apparently overlooked that long before the search warrant was
applied for, to be specific on April 15, 1968, the misdeclaration and underpayment was already noted and that
thereafter on April 24, 1968, private respondent himself agreed to make good the further amount due but not in the
sum demanded.
As the car was kept in a dwelling house, petitioner through two of his officers in the Customs Police Service applied
for and was able to obtain the search warrant. Had there been no such move on the part of petitioner, the duties
expressly enjoined on him by law assess and collect all lawful revenues, to prevent and suppress smuggling and
other frauds and to enforce tariff and customs law would not have been performed.
While therefore, it is to be admitted that his warrant of seizure and detention came later than the search warrant,
there were indubitable facts in existence at that time to call for its issuance. Certainly there was probable cause.
There was evidently need for the issuance of a search warrant. It ought not to have been thereafter quashed.

DIZON VS CASTRO, April 11, 1985

PEOPLE VS VELOSO, 48 PHIL 169


parliamentary club - JOHN DOE WARRANTS Valid IF the best description possible is given in the arrest warrant
it must be sufficient to indicate clearly on whom it is to be served by stating his occupation, personal appearance
or peculiarities, place of residence or other circumstances which he may be identified
In May, 1923, the building located at No. 124 Calle Arzobispo, City of Manila, was used by an organization known
as the Parliamentary Club. Jose Ma. Veloso was at that time a member of the House of Representative of the
Philippine Legislature. He was also the manager of the club.-The police of Manila had reliable information that the
so-called Parliamentary Club was nothing more than a gambling house. Indeed, on May 19, 1923, J. F. Townsend,
the chief of the gambling squad, had been to the club and verified this fact. As a result, on May 25, 1923, Detective
Andres Geronimo of the secret service of the City of Manila, applied for, and obtained a search warrant from Judge
Garduo of the municipal court. Thus provided, the police attempted to raid the Parliamentary Club a little after
three in the afternoon of the date above- mentioned. They found the doors to the premises closed and barred.
Accordingly, one band of police including policeman Rosacker, ascended a telephone pole, so as to enter a window
of the house. Other policemen, headed by Townsend, broke in the outer door.-Once inside the Parliamentary Club,
nearly fifty persons were apprehended by the police. One of them was the defendant Veloso. Veloso asked
Townsend what he wanted, and the latter showed him the search warrant. Veloso read it and told Townsend that
he was Representative Veloso and not John Doe, and thatthe police had no right to search the house. Townsend
answered that Veloso was considered as John Doe. As Veloso's pocket was bulging, as if it contained gambling
utensils, Townsend required Veloso to show him the evidence of the game. About five minutes was consumed in
conversation between the policemen and the accused the policemen insisting on searching Veloso, and Veloso
insisting in his refusal to submit to the search.-At last the patience of the officers was exhausted. So policeman
Rosacker took hold of Veloso only to meet with his resistance. Veloso bit Rosacker in the right forearm, and gave
him a blow in another part of the body, which injured the policeman quite severely. Through the combined efforts of
Townsend and Rosacker, Veloso was finally laid down on the floor, and long sheets of paper, of reglas de monte,
cards, cardboards, and chips were taken from his pockets.-All of the persons arrested were searched and then
conducted to the patrol wagons. Veloso again refusedto obey and shouted offensive epithets against the police
department. It was necessary for the policemen to conduct him downstairs. At the door, Veloso resisted so
tenaciously that three policemen were needed to place him in the patrol wagon.
ISSUE: WON the search warrant and the arrest of Veloso was valid.
RULING: Yes. It is provided, among other things, in the Philippine Code on Criminal Procedure that a search
warrant shall not issue except for probable cause and upon application supported by oath particularly describing
the place to be searched and the person of thing to be seized. The name and description of the accused should be
inserted in the body of the warrant and where the name is unknown there must be such a description of the person
accused as will enable the officer to identify him when found. A warrant for the apprehension of a person whose
true name is unknown, by the name of "John Doe" or Richard Roe," "whose other or true name in unknown," is
void, without other and further descriptions of the person to be apprehended, and such warrant will not justify the
officer in acting under it. Such a warrant must, in addition, contain the best descriptio personae possible to be
obtained of the person or persons to be apprehended, and this description must be sufficient to indicate clearly the
proper person or persons upon whom the warrant is to be served; and should state his personal appearance and
peculiarities, give his occupation and place of residence, and any other circumstances by means of which he can
be identified. In the first place, the affidavit for the search warrant and the search warrant itself described the
building to be searched as "the building No. 124 Calle Arzobispo, City of Manila, Philippine Islands." This, without
doubt, was a sufficient designation of the premises to be searched. As the search warrant stated that John Doe had
gambling apparatus in his possession in the building occupied by him at No. 124 Calle Arzobispo, City of Manila,
and as this John Doe was Jose Ma. Veloso, the manager of the club, the police could identify John Doe as Jose
Ma. Veloso without difficulty

LUI VS MATILLANO, May 27, 2004


Right against unreasonable searches and seizures; Mission Order does not authorize an illegal search; Waiver of
the right against an unreasonable search and seizure.
In search of the allegedly missing amount of P45,000 owned by the employer, the residence of a relative of the
suspect was forcibly open by the authorities by kicking the kitchen door to gain entry into the house. Thereafter,
they confiscated different personal properties therein which were allegedly part of those stolen from the employer.
They were in possession of a mission order but later on claimed that the owner of the house gave his consent to
the warrantless search. Are the things admissible in evidence? Can they be sued for damages as a result of the
said warrantless search and seizure?

HELD: The right against unreasonable searches and seizures is a personal right which may be waived expressly or
impliedly. BUT A WAIVER BY IMPLICATION CANNOT BE PRESUMED. There must be clear and convincing
evidence of an actual intention to relinquish the right. There must be proof of the following:
a. that the right exists;
b. that the person involved had knowledge, either constructive or actual, of the existence of said right;
c. that the said person had an actual intention to relinquish the right.
Finally, the waiver must be made voluntarily, knowingly and intelligently in order that the said is to be valid.
The search was therefore held illegal and the members of the searching party held liable for damages in
accordance with the doctrine laid down in Lim vs. Ponce de Leon and MHP Garments vs. CA.

TWO KINDS OF PROBABLE CAUSE


1. The executive determination of probable cause on whether a crime was allegedly committed and file a
criminal case in court; and
2. Judicial determination of probable cause for the issuance of a warrant of arrest.

MICROSOFT CORPORATION VS FARAJALLAH, GR 205800

LEVISTE VS ALAMEDA, GR 182677


There is no waiver of the illegality of the arrest if the accused refused to enter a voluntary plea during arraignment
and continued to question the validity of the preliminary investigation which resulted in the filing of the information in
court.

BORLONGAN VS PENA AND LIMSIACO, GR 143591


Respondent Magdaleno Pea instituted a civil case for recovery of agents compensation and expenses, damages,
and attorneys fees, against Urban Bank and the petitioners, before the Regional Trial Court (RTC) of Negros
Occidental, Bago City.- Respondent anchored his claim for compensation on the contract of agency, allegedly
entered into with the petitioners wherein the former undertook to perform such acts necessary to prevent any
intruder and squatter from unlawfully occupying Urban Banks property located along Roxas Boulevard, Pasay City.
Petitioners filed a MD arguing that they never appointed the respondent as agent or counsel.
Attached to the MD were the following documents:
1. A letter dated December 19, 1994 signed by Herman Ponce and Julie Abad on behalf of Isabela Sugar
Company, Inc. (ISCI), the original owner of the subject property;
2. An unsigned letter dated December 7, 1994 addressed to Corazon Bejasa from Marilyn G. Ong;
3. A letter dated December 9, 1994 addressed to Teodoro Borlongan and signed by Marilyn G. Ong; and
4. A Memorandum dated November 20, 1994 from Enrique Montilla III.
The above stated documents were presented in an attempt to show that the respondent was appointed as agent by
ISCI and not by Urban Bank or by the petitioners.
Respondent Pea filed his Complaint-Affidavit with the Office of the City Prosecutor, Bago City. He claimed that
said documents were falsified because the alleged signatories did not actually affix their signatures, and the
signatories were neither stockholders nor officers and employees of ISCI. Worse, petitioners introduced said
documents as evidence before the RTC knowing that they were falsified.
City Prosecutors Report (Sept 23, 1998): In the report, the Prosecutor concluded that the petitioners were probably
guilty of four (4) counts of the crime of Introducing Falsified Documents penalized by the second paragraph of
Article 172 of the Revised Penal Code (RPC). The City Prosecutor concluded that the documents were falsified
because the alleged signatories untruthfully stated that ISCI was the principal of the respondent; that petitioners
knew that the documents were falsified considering that the signatories were mere dummies; and that the
documents formed part of the record of Civil Case No. 754 where they were used by petitioners as evidence in
support of their motion to dismiss, adopted in their answer and later, in their Pre-Trial Brief. Subsequently, the
corresponding Informations were filed with the Municipal Trial Court in Cities (MTCC), Bago City. The cases were
docketed as Criminal Cases Nos. 6683, 6684, 6685, and 6686. Thereafter, Judge Primitivo Blanca issued the
warrants for the arrest of the petitioners.
Petitioners (Oct `1, 1998) filed an Omnibus MQ: They insist that they were denied due process because of the nonobservance of a proper procedure on preliminary investigation prescribed in the Rules of Court; since no such
counter-affidavit and supporting documents were submitted by the petitioners, the trial judge merely relied on the
complaint-affidavit and attachments of the respondent in issuing the warrants of arrest, also in contravention of the
Rules. Moreover they claim that the respondents affidavit was not based on the latters personal knowledge and

therefore should not have been used by the court in determining probable cause.-On the same day that the
Omnibus MQ was filed, the petitioners posted bail. Their bail bonds expressly provided that they do not intend to
waive their right to question the validity of their arrest. On the date of arraignment, the petitioners refused to enter
their plea, for the obvious reason that the legality of their information and their arrest was yet to be settled by the
court.-MTCCs answer (in response to Omnibus MQ filed by petitioners): They upheld the validity of the warrant of
arrest, saying that it was issued in accordance with the Rules. Besides, (according to the MTCC) petitioners could
no longer question the validity of the warrant since they already posted bail.
ISSUES:
1) WON petitioners were deprived of their right to due process of law because of the denial of their right to
preliminary investigation and to submit their counter-affidavit;
2) WON the Informations charging the petitioners were validly filed and the warrants for their arrest were
properly issued;
3) WON this Court can, itself, determine probable cause; and
4) WON the petitioners posting a bail constitutes a waiver of their right to question the validity of their arrest.
HELD: Petition granted; MTCC is ordered to dismiss criminal cases against petitioners.
For issues numbered 1 and 3:
The following sections of Rule 112 of the 1985 Rules of Criminal Procedure are relevant to the aforesaid issues:
SECTION 1. Definition. Preliminary investigation is an inquiry or proceeding for the purpose of
determining whether there is sufficient ground to engender a well-founded belief that a crime
cognizable by the Regional Trial Court has been committed and that the respondent is probably
guilty thereof, and should be held for trial.
SEC. 3. Procedure. Except as provided for in Section 7 hereof, no complaint or information for an
offense cognizable by the Regional Trial Court shall be filed without a preliminary investigation
having been first conducted in the following manner:(a) The complaint shall state the known
address of the respondent and be accompanied by affidavits of the complainant and his witnesses
as well as other supporting documents, in such number of copies as there are respondents, plus
two (2) copies of the official file. The said affidavits shall be sworn to before any fiscal, state
prosecutor or government official authorized to administer oath, or, in their absence or
unavailability, a notary public, who must certify that he personally examined the affiants and that he
is satisfied that they voluntarily executed and understood their affidavits. SEC. 9. Cases not
falling under the original jurisdiction of the Regional Trial Courts not covered by the Rule on
Summary Procedure. (a) Where filed with the fiscal. If the complaint is filed directly with the
fiscal or state prosecutor, the procedure outlined in Section 3 (a) of this Rule shall be observed. The
Fiscal shall take appropriate action based on the affidavits and other supporting documents
submitted by the complainant.
Records show that the prosecutor relied merely on the affidavits submitted by the complainant and did not require
the petitioners to submit their answer. He should not be faulted for doing such as this is sanctioned by the rules.
Moreover, he is not mandated to require the submission of counter-affidavits. Probable cause may then be
determined on the basis alone of the affidavits and supporting documents of the complainant, without infringing on
the constitutional rights of the petitioners.-Regarding the issuance of the warrant of arrest, petitioners contend that
the warrants were illegally issued as they were solely based on the affidavits of the complainant. Section 2 of Article
III of the Constitution underscores the exclusive and personal responsibility of the issuing judge to satisfy himself of
the existence of probable cause. But the judge is not required to personally examine the complainant and his
witnesses. Following established doctrine and procedure, he shall (1) personally evaluate the report and the
supporting documents submitted by the prosecutor regarding the existence of probable cause, and on the basis
thereof, he may already make a personal determination of the existence of probable cause; and (2) if he is not
satisfied that probable cause exists, he may disregard the prosecutors report and require the submission of
supporting affidavits of witnesses to aid him in arriving at a conclusion as to the existence of probable cause. There
is no provision or procedural rule which makes the submission of counter-affidavits mandatory before the judge
could determine probable cause.
For issue number 2:
For the issuance of a warrant of arrest, probable cause has been defined as the existence of such facts and
circumstances that would lead a reasonably discreet and prudent person to believe that an offense has been
committed by the person sought to be arrested. It is one of the requisites for a warrant of arrest to be valid.
On the basis of the above-stated documents (in the facts) and on the strength of the affidavit executed by the
respondent, the prosecutor concluded that probable cause exists. These same affidavit and documents were used
by the trial court in issuing the warrant of arrest.
The SC finds the complaint-affidavit and attachments insufficient to support the existence of probable cause. The
respondents claims of the falsity of the documents were mere assertions.

It must be emphasized that the affidavit of the complainant, or any of his witnesses, shall allege facts within their
(affiants) personal knowledge. The allegation of the respondent that the signatures were falsified does not qualify
as personal knowledge. Nowhere in said affidavit did respondent state that he was present at the time of the
execution of the documents. Neither did he claim that he was familiar with the signatures of the signatories. He
simply made a bare assertion.
A finding of probable cause need not be based on clear and convincing evidence, or on evidence beyond
reasonable doubt. It does not require that the evidence would justify conviction. Nonetheless, although the
determination of probable cause requires less than evidence which would justify conviction, it should at least be
more than mere suspicion. While probable cause should be determined in a summary manner, there is a need to
examine the evidence with care to prevent material damage to a potential accuseds constitutional right to liberty
and the guarantees of freedom and fair play, and to protect the State from the burden of unnecessary expenses in
prosecuting alleged offenses and holding trials arising from false, fraudulent or groundless charges.

ROBERTS VS COURTS OF APPEALS, 254 SCRA 307


Petitioners, who are corporate officers and members of the Board of Pepsi Cola Products Phils., Inc. were
prosecuted in connection with the Pepsi Number Fever promotion by handlers of the supposedly winning 349
Pepsi crowns. Of the four cases filed against the petitioners, probable cause was found by the investigating
prosecutor only for the crime of estafa, but not for the other alleged offenses.
On 12 April 1993, the information was filed with the trial court without anything accompanying it. A copy of the
investigating prosecutors Joint Resolution was forwarded to and received by the trial court only on 22 April 1993.
However, no affidavits of the witnesses, transcripts of stenographic notes of the proceedings during the preliminary
investigation, or other documents submitted in the course thereof were found in the records of the case as of 19
May 1993.
On 15 April 1993, petitioners Roberts, et al. filed a petition for review to the Department of Justice seeking the
reversal of the finding of probable cause by the investigating prosecutor. They also moved for the suspension of the
proceedings and the holding in abeyance of the issuance of warrants of arrest against them. Meanwhile, the public
prosecutor also moved to defer the arraignment of the accused-appellants pending the final disposition of the
appeal to the Department of Justice.
On 17 May 1993, respondent Judge Asuncion issued the challenged order (1) denying, on the basis of Crespovs.
Mogul, the foregoing motions respectively filed by the petitioners and the public prosecutor, and directing the
issuance of the warrants of arrest after June 1993 and setting the arraignment on 28 June 1993. In part,
respondent judge stated in his order that since the case is already pending in this Court for trial, following whatever
opinion the Secretary of Justice may have on the matter would undermine the independence and integrity his court.
To justify his order, he quoted the ruling of the Supreme Court in Crespo, which stated:
In order therefor to avoid such a situation whereby the opinion of the Secretary of Justice who reviewed the action
of the fiscal may be disregarded by the trial court, the Secretary of Justice should, as far as practicable, refrain from
entertaining a petition for review or appeal from the action of the fiscal, when the complaint or information has
already been filed in Court. The matter should be left entirely for the determination of the Court.
Petitioners went to the Court of Appeals (CA), arguing that the respondent judge had not the slightest basis at all
for determining probable cause when he ordered the issuance of warrants of arrest. After finding that a copy of the
public prosecutors Joint Resolution had in fact been forwarded to, and received by, the trial court on 22 April 1993,
the CA denied petitioners application for writ of preliminary injunction. The CA ruled that the Joint Resolution was
sufficient in itself to have been relied upon by respondent Judge in convincing himself that probable cause indeed
exists for the purpose of issuing the corresponding warrants of arrest and that the mere silence of the records or
the absence of any express declaration in the questioned order as to the basis of such finding does not give rise to
an adverse inference, for the respondent Judge enjoys in his favor the presumption of regularity in the performance
of his official duty. Roberts, et al. sought reconsideration, but meanwhile, the DOJ affirmed the finding of probable
cause by the investigating prosecutor. The CA therefore dismissed the petition for mootness.
ISSUES:
1. Did Judge Asuncion commit grave abuse of discretion in denying, on the basis of Crespo vs. Mogul, the
motions to suspend proceedings and hold in abeyance the issuance of warrants of arrest and to defer
arraignment until after the petition for review filed with the DOJ shall have been resolved?
2. Did Judge Asuncion commit grave abuse of discretion in ordering the issuance of warrants of arrest without
examining the records of the preliminary investigation?
3. May the Supreme Court determine in this [sic] proceedings the existence of probable cause either for the
issuance of warrants of arrest against the petitioners or for their prosecution for the crime of estafa?
HELD: The Court, in a 7-5-2 vote, GRANTED the petition. It SET ASIDE the decision and resolution of the CA, the
resolutions of the DOJ 349 Committee, and the order of respondent judge.

1. YES, Judge Asuncion committed grave abuse of discretion in denying, on the basis of Crespo vs. Mogul,
the motions to suspend proceedings and hold in abeyance the issuance of warrants of arrest and to defer
arraignment until after the petition for review filed with the DOJ shall have been resolved.
There is nothing in Crespo vs. Mogul which bars the DOJ from taking cognizance of an appeal, by way of a
petition for review, by an accused in a criminal case from an unfavorable ruling of the investigating
prosecutor. It merely advised the DOJ to, as far as practicable, refrain from entertaining a petition for
review or appeal from the action of the fiscal, when the complaint or information has already been filed in
Court.
Whether the DOJ would affirm or reverse the challenged Joint Resolution is still a matter of guesswork.
Accordingly, it was premature for respondent Judge Asuncion to deny the motions to suspend proceedings
and to defer arraignment on the following grounds:
This case is already pending in this Court for trial. To follow whatever opinion the Secretary of Justice may
have on the matter would undermine the independence and integrity of this Court. This Court is still
capable of administering justice.
The real and ultimate test of the independence and integrity of this court is not the filing of the
aforementioned motions [to suspend proceedings and issuance of warrants of arrest and to defer
arraignment] at that stage but the filing of a motion to dismiss or to withdraw the information on the basis of
a resolution of the petition for review reversing the Joint Resolution of the investigating prosecutor.
However, once a motion to dismiss or withdraw the information is filed the trial judge may grant or deny it,
not out of subservience to the Secretary of Justice, but in faithful exercise of judicial prerogative.
2. YES, Judge Asuncion committed grave abuse of discretion in ordering the issuance of warrants of arrest
without examining the records of the preliminary investigation.
The teachings then of Soliven, Inting, Lim, Allado, and Webb reject the proposition that the investigating
prosecutors certification in an information or his resolution which is made the basis for the filing of the
information, or both, would suffice in the judicial determination of probable cause for the issuance of a
warrant of arrest. In Webb, this Court assumed that since the respondent Judges had before them not only
the 26-page resolution of the investigating panel but also the affidavits of the prosecution witnesses and
even the counter-affidavits of the respondents, they (judges) made personal evaluation of the evidence
attached to the records of the case.
In this case, nothing accompanied the information upon its filing on 12 April 1993 with the trial court. A copy
of the Joint Resolution was forwarded to, and received by, the trial court only on 22 April 1993. And as
revealed by the certification of respondent judges clerk of court, no affidavits of the witnesses, transcripts
of stenographic notes of the proceedings during the preliminary investigation, or other documents
submitted in the course thereof were found in the records of this case as of 19 May 1993. Clearly, when
respondent Judge Asuncion issued the assailed order of 17 May 1993 directing, among other things, the
issuance of warrants of arrest, he had only the information, amended information, and Joint Resolution as
bases thereof. He did not have the records or evidence supporting the prosecutor's finding of probable
cause. And strangely enough, he made no specific finding of probable cause; he merely directed the
issuance of warrants of arrest after June 21, 1993. It may, however, be argued that the directive
presupposes a finding of probable cause. But then compliance with a constitutional requirement for the
protection of individual liberty cannot be left to presupposition, conjecture, or even convincing logic.
3. NO, the Supreme Court MAY NOT determine in this [sic] proceedings the existence of probable cause
either for the issuance of warrants of arrest against the petitioners or for their prosecution for the crime of
estafa.
Ordinarily, the determination of probable cause is not lodged with this Court. Its duty in an appropriate case
is confined to the issue of whether the executive or judicial determination, as the case may be, of probable
cause was done without or in excess of jurisdiction or with grave abuse of discretion amounting to want of
jurisdiction. This is consistent with the general rule that criminal prosecutions may not be restrained or
stayed by injunction, preliminary or final.
There are, however, exceptions to the foregoing rule. But the Court refused to reevaluate the evidence to
determine if indeed there is probable cause for the issuance of warrants of arrest in this case. For the
respondent judge did not, in fact, find that probable cause exists, and if he did he did not have the basis
therefor. Moreover, the records of the preliminary investigation in this case are not with the Court. They
were forwarded by the Office of the City Prosecutor of Quezon City to the DOJ in compliance with the
latter's 1st Indorsement of 21 April 1993. The trial court and the DOJ must be required to perform their duty.

SKECHERS USA, INC. VS PACIFIC, GR 164321

MILLER VS SECRETARY OF JUSTICE, GR 165412


Probable cause; definition. Probable cause is defined as the existence of such facts and circumstances as would
excite the belief in a reasonable mind, acting on the facts within the knowledge of the prosecutor, that the person
charged was guilty of the crime for which he was prosecuted.
Probable cause; determination. To determine the existence of probable cause, there is need to conduct preliminary
investigation. A preliminary investigation constitutes a realistic judicial appraisal of the merits of a case. Its purpose
is to determine whether (a) a crime has been committed; and (b) whether there is a probable cause to believe that
the accused is guilty thereof. It is a means of discovering which person or persons may be reasonably charged with
a crime. It is well-settled that the determination of probable cause for the purpose of filing an information in court is
an executive function which pertains at the first instance to the public prosecutor and then to the Secretary of
Justice. The Secretary of Justice may reverse or modify the resolution of the prosecutor, after which he shall direct
the prosecutor concerned either to file the corresponding information without conducting another preliminary
investigation, or to dismiss or move for dismissal of the complaint or information with notice to the parties.

PEOPLE VS DEL ROSARIO, July 10, 1994


After the informant was given by the police the amount of P100.00, he went to buy marijuana from the accused
then return to the police headquarters with said article. Thereafter, the policemen went to arrest the accused
without warrant. The arrest is not valid since it does not fall under Section 5 Rule 113.
Likewise, after securing a search warrant authorizing the seizure of shabu and its paraphernalia and instead, an
unlicensed firearm was seized instead, said gun is inadmissible in evidence.
The policemen gave P100.00 to the informant to buy marijuana from the accused. After he returned with the
marijuana, the policemen went to arrest the accused. The arrest is not valid since the same does not fall under
Section 5 Rule 113. They have no personal knowledge.
Accused was charged and convicted by the trial court of illegal possession of firearms and illegal possession and
sale of drugs, particularly methamphetamine or shabu. After the issuance of the search warrant, which authorized
the search and seizure of an undetermined quantity of methamphetamine and its paraphernalias, an entrapment
was planned that led to the arrest of del Rosario and to the seizure of the shabu, its paraphernalias and of a .22
caliber pistol with 3 live ammunition.
ISSUE: Whether or Not the seizure of the firearms was proper.
HELD: No, Section 2, Article III of the constitution specifically provides that a search warrant must particularly
describe the things to be seized. In herein case, the only objects to be seized that the warrant determined was the
methamphetamine and the paraphernalias therein. The seizure of the firearms was unconstitutional.
Wherefore the decision is reversed and the accused is acquitted.

DELOS SANTOS VS MONTESA, 247 SCRA 85

LIM VS FELIX
Warrant of arrest issued by the RTC Makati based on the Resolution and the Information filed by the Fiscal is not
valid since the record of the case with the affidavits of witnesses is still in Masbate.
Certification by the fiscal of the existence of probable cause does not bind the judge. Preliminary inquiry
determines probable cause for the issuance of a search warrant (prosecutor); Preliminary examination (judge)
investigation for the determination of a probable cause for the issuance of a warrant of arrest; preliminary
investigation proper ascertains whether the offender should be held for trial or be released.
Petitioners are suspects of the slaying of congressman Moises Espinosa, Sr. and three of his security escorts and
the wounding of another. They were initially charged, with three others, with the crime of multiple murder with
frustrated murder. After conducting a preliminary investigation, a warrant of arrest was issued on July 31, 1989. Bail
was fixed at P200,000.
On September 22, 1989, Fiscal Alfane, designated to review the case, issued a Resolution affirming the finding of a
prima facie case against the petitioners but ruled that a case of Murder for each of the killing of the four victims and
a physical injuries case for inflicting gunshot wound on the survivor be filled instead against the suspects.
Thereafter, four separate informations to that effect were filed with the RTC of Masbate with no bail recommended.
On November 21, 1989, a motion for change of venue, filed by the petitioners was granted by the SC. It ordered
that the case may be transferred from the RTC of Masbate to the RTC of Makati.

Petitioners then moved that another hearing be conducted to determine if there really exists a prima facie case
against them in the light of documents showing recantations of some witnesses in the preliminary investigation.
They likewise filed a motion to order the transmittal of initial records of the preliminary investigation conducted by
the municipal judge of Barsaga of Masbate. These motions were however denied by the court because the
prosecution had declared the existence of probable cause, informations were complete in form in substance and
there was no defect on its face. Hence it found it just and proper to rely on the prosecutors certification in each
information.
ISSUE: Whether or not a judge may issue a warrant of arrest without bail by simply relying on the prosecutions
certification and recommendation that a probable cause exists?
HELD:
1. The judge committed a grave abuse of discretion. In the case of Placer vs. Villanueva, the sc ruled that a
judge may rely upon the fiscals certification of the existence of a probable cause and on the basis thereof,
issue a warrant of arrest. However, the certification does not bind the judge to come out with the warrant of
arrest. This decision interpreted the search and seizure provision of the 1973 Constitution. Under this
provision, the judge must satisfy himself of the existence of probable cause before issuing a warrant of order
of arrest. If on the face of information, the judge finds no probable cause, he may disregard the fiscals
certification and require the submission of the affidavits of witness to aid him at arriving at a conclusion as to
the existence of a probable cause. This has been the rule since U.S vs. Ocampo and Amarga vs. Abbas.
2. In the case of Soliven vs. Makasiar, decided under the 1987 Constitution, the Court noted that the addition of
the word personally after the word determined and the deletion of the grant of authority by the 1973
Constitution to issue warrants to other respondent officers as to may be authorized by law does not require
the judge to personally examine the complainant and his witness in his determination of probable cause for
the issuance of a warrant of arrest.What the Constitution underscores is the exclusive and personal
responsibility of the issuing judge to satisfy himself of the existence of probable cause. Following established
doctrine and procedures, he shall:
(1) personally evaluate the reports and the supporting documents submitted by the fiscal regarding the
existence of probable cause and, on the basis thereof, issue a warrant of arrest;
(2) If on the basis thereof he finds no probable cause, he may disregard the fiscals report and require the
submission of supporting affidavits of witnesses to aid him in arriving at a conclusion as to the
existence of probable cause.
3. The case of People vs. Honorable Enrique B. Inting reiterates the following doctrines:
(1) The determination of probable cause is a function of the judge. It is not for the Provincial Fiscal or
Prosecutor nor for the Election Supervisor to ascertain. Only the judge alone makes this detemination.
(2) The preliminary inquiry made by the prosecutor does not bind the judge. It merely assist him to make
the determination of probable cause. The judge does not have to follow what the prosecutors present
to him. By itself, the prosecutors certification of probable cause is ineffectual. It is the report, the
affidavits, the transcripts of stenographic notes, and all other supporting documents behind the
prosecutors certification which are material in assisting the judge to make his determination.
(3) Preliminary inquiry should be distinguished from the preliminary investigation proper. While the former
seeks to determine probable cause for the issuance of warrant of arrest, the latter ascertains whether
the offender should be held for trial or be released.
4. In the case of Castillo vs. Villaluz, the court ruled that judges of RTC no longer have authority to conduct
preliminary investigations: This authority was removed from them by the 1985 Rules on Criminal Procedure,
effective on January 1, 1985.
5. In the present case, the respondent judge relies solely on the certification of the prosecutor. Considering that
all the records of the investigation are in Masbate, he has not personally determined the existence of
probable cause. The determination was made by the provincial prosecutor. The constitutional requirement
had not been satisfied.
The records of the preliminary investigation conducted by the Municipal Court of Masbate and reviewed by
the respondent Fiscal were still in Masbate when the respondent Fiscal issued the warrant of arrest against
the petitioners. There was no basis for the respondent judge to make his personal determination regarding
the existence of probable cause from the issuance of warrant of arrest as mandated by the Constitution. He
could not have possibly known what has transpired in Masbate as he had nothing but a certification. Although
the judge does not have to personally examine the complainant and his witnesses (for the prosecutor can
perform the same functions as commissioner for taking of evidence) there should be a report and necessary
documents supporting the Fiscals bare certification. All of these should be before the judge.

AMARGA VS ABBAS, 98 PHIL. 739


Natalio P. Amarga, the Provincial Fiscal of Sulu, filed in the Court of First Instance of Sulu an information for murder
(criminal case 1131, People of the Philippines vs. Madpirol, Awadi, Rajah, Sali, Insa and Maharajah Bapayani). At
the foot of the information, Amarga certified under oath that "he has conducted the necessary preliminary
investigation pursuant to the provisions of Republic Act 732." As the only supporting affidavit was that of Iman Hadji
Rohmund Jubair, to the effect that the latter "was told that the deceased was shot and killed by three persons
named: Hajirul Appang, Rajah Appang and Awadi Bagali," and Amarga had failed or refused to present other
evidence sufficient to make out a prima facie case, Judge Macapanton Abbas (CFI of Sulu) issued an order,
dismissing the case without prejudice to reinstatement should the provincial fiscal support his information with
record of his investigation which in the opinion of the court may support a prima facie case. Amarga instituted a
petition for certiorari and mandamus before the Supreme Court.
ISSUE: Whether the preliminary investigation conducted by Amarga dispenses with the judges duty to determine
probable cause exists before issuing the corresponding warrant of arrest.
HELD: Section 1, paragraph 3, of Article III of the Constitution provides that "no warrant shall issue but upon
probable cause, to be determined by the judge after examination under oath or affirmation of the complainant and
the witnesses he may produce." The question whether "probable cause" exists or not must depend upon the
judgment and discretion of the judge or magistrate issuing the warrant. If he is satisfied that "probable cause" exists
from the facts stated in the complaint, made upon the investigation by the prosecuting attorney, then his conclusion
is sufficient upon which to issue the warrant for arrest. He may, however, if he is not satisfied, call such witnesses
as he may deem necessary before issuing the warrant. The issuance of the warrant of arrest is prima facie
evidence that, in his judgment at least, there existed "probable cause" for believing that the person against whom
the warrant is issued is guilty of the crime charged. The preliminary investigation conducted by Amarga under
Republic Act 732 which formed the basis for the filing in the Court of First Instance of Sulu of criminal case 1131
does not dispense with the judge's duty to exercise his judicial power of determining, before issuing the
corresponding warrant of arrest, whether or not probable cause exists therefor. The Constitution vests such power
in the judge who, however, may rely on the facts stated in the information filed after preliminary investigation by the
prosecuting attorney.
TH

20 CENTURY FOX VS COURT OF APPEALS, 164 SCRA 655


In a letter-complaint dated 26 August 1985, 20th Century Fox Film Corporation through counsel sought the National
Bureau of Investigation's (NBI) assistance in the conduct of searches and seizures in connection with the NBI's
anti-film piracy campaign. Specifically, the letter-complaint alleged that certain videotape outlets all over Metro
Manila are engaged in the unauthorized sale and renting out of copyrighted films in videotape form which constitute
a flagrant violation of Presidential Decree 49 (Decree on the Protection of Intellectual Property). Acting on the lettercomplaint, the NBI conducted surveillance and investigation of the outlets pinpointed by the film corporation and
subsequently filed 3 applications for search warrants against the video outlets owned by Eduardo M. Barreto, Raul
Sagullo, and Fortune Ledesma. The applications were consolidated and heard by the Regional Trial Court (RTC) of
Makati, Branch 132. On 4 September 1985, the lower court issued the desired search warrants, describing the
articles sought to be seized as"(c) Television sets, Video Cassettes Recorders, rewinders, tape head cleaners,
accessories, equipment and other machines used or intended to be used in the unlawful reproduction, sale,
rental/lease, distribution of the above-mentioned video tapes which she is keeping and concealing in the premises
above-described.". Armed with the search warrants, the NBI accompanied by the film corporation's agents, raided
the video outlets and seized the items described therein. An inventory of the items seized was made and left with
Barreto, et. al. Acting on a motion to lift search warrants and release seized properties filed by Barreto, et. al., the
lower court issued an order dated 8 October 1985, lifting the 3 search warrants issued earlier against them by the
court, due to the failure of the NBI to deliver the articles to the Court, and thus ordered the return of the articles to
their respective owners. The lower court denied a motion for reconsideration filed by the film corporation in its order
dated 2 January 1986. The film corporation filed a petition for certiorari with the Court of Appeals to annul the
orders of the lower court. The petition was dismissed. The 20th Century Fox Film Corporation filed the petition for
review with the Supreme Court.
ISSUE: Whether the inclusion of certain articles of property which are usually connected to legitimate business, and
not involving piracy of intellectual property or infringement of copyright laws, renders the warrant to be
unreasonable.
HELD: Television sets, video cassette recorders, rewinders and tape cleaners are articles which can be found in a
video tape store engaged in the legitimate business of lending or renting out betamax tapes. In short, these articles
and appliances are generally connected with, or related to a legitimate business not necessarily involving piracy of
intellectual property or infringement of copyright laws. Hence, including these articles without specification and/or
particularity that they were really instruments in violating an Anti-Piracy law makes the search warrant too general
which could result in the confiscation of all items found in any video store. In fact, this actually happened in the

present case. Although the applications and warrants themselves covered certain articles of property usually found
in a video store, the Court believes that the search party should have confined themselves to articles that are
according to them, evidence constitutive of infringement of copyright laws or the piracy of intellectual property, but
not to other articles that are usually connected with, or related to, a legitimate business, not involving piracy of
intellectual property, or infringement of copyright laws. So that a television set, a rewinder, and a whiteboard listing
Betamax tapes, video cassette cleaners video cassette recorders as reflected in the Returns of Search Warrants,
are items of legitimate business engaged in the video tape industry, and which could not be the subject of seizure.
The applicant and his agents therefore exceeded their authority in seizing perfectly legitimate personal property
usually found in a video cassette store or business establishment. The search and seizure is unreasonable.

QUINTERO VS NBI, 162 SCRA 467


Searching parties searched different rooms simultaneously thereby resulting in no witnesses in the other rooms
searched, the search is not valid.

PRESIDENTIAL ANTI-DOLLAR SALTING TASK FORCE VS COURT OF APPEALS

SOLIVEN & BELTRAN VS MAKASIAR, NOVEMBER 18, 1988 (Note: widely criticized)
The word personally after the word determined does not necessarily mean that the judge should examine the
complainant and his witnesses personally before issuing the search warrant or warrant of arrest but the exclusive
responsibility on the part of said judge to satisfy himself of the existence of probable cause. As such, there is no
need to examine the complainant and his witnesses face to face. It is sufficient if the judge is convinced of the
existence of probable cause upon reading the affidavits or deposition of the complainant and his witnesses.
There is no need for the judge to examine the complainant and the witnesses face to face in order to determine
probable cause. It is enough that he shall personally examine the affidavits of the complainant and the witnesses
and if he is convinced that there is probable cause, he can validly issue the warrant of arrest
In these consolidated cases, three principal issues were raised: (1) whether or not petitioners were denied due
process when informations for libel were filed against them although the finding of the existence of a prima facie
case was still under review by the Secretary of Justice and, subsequently, by the President; and (2) whether or not
the constitutional rights of Beltran were violated when respondent RTC judge issued a warrant for his arrest without
personally examining the complainant and the witnesses, if any, to determine probable cause. Subsequent events
have rendered the first issue moot and academic. On March 30, 1988, the Secretary of Justice denied petitioners'
motion for reconsideration and upheld the resolution of the Undersecretary of Justice sustaining the City Fiscal's
finding of a prima facie case against petitioners. A second motion for reconsideration filed by petitioner Beltran was
denied by the Secretary of Justice on April 7, 1988. On appeal, the President, through theExecutive Secretary,
affirmed the resolution of the Secretary of Justice on May 2, 1988. The motion for reconsideration was denied by
the Executive Secretary on May 16, 1988. With these developments, petitioners' contention that they have been
denied the administrative remedies available under the law has lost factual support.
ISSUES: (1) Whether or Not petitioners were denied due process when informations for libel were filed against
them although the finding of the existence of a prima facie case was still under review by the Secretary of Justice
and, subsequently, by the President.
(2) Whether or Not the constitutional rights of Beltran were violated when respondent RTC judge issued a warrant
for his arrest without personally examining the complainant and the witnesses, if any, to determine probable cause
HELD: With respect to petitioner Beltran, the allegation of denial of due process of law in the preliminary
investigation is negated by the fact that instead of submitting his counter- affidavits, he filed a "Motion to Declare
Proceedings Closed," in effect waiving his right to refute the complaint by filing counter-affidavits. Due process of
law does not require that the respondent in a criminal case actually file hiscounter-affidavits before the preliminary
investigation is deemed completed. All that is required is that the respondent be given theopportunity to submit
counter-affidavits if he is so minded.
The second issue, raised by petitioner Beltran, calls for an interpretation of the constitutional provision on the
issuance of warrants of arrest. The pertinent provision reads:
Art. III, Sec. 2. The right of the people to be secure in their persons, houses, papers and effects
against unreasonable searches and seizures of whatever nature and for any purpose shall be
inviolable, and no search warrant or warrant of arrest shall issue except upon probable cause to be
determined personally by the judge after examination under oath or affirmation of the complainant
and the witnesses he may produce, and particularly describing the place to be searched and the
persons or things to be seized.

The addition of the word "personally" after the word "determined" and the deletion of the grant of authority by the
1973 Constitution to issue warrants to "other responsible officers as may be authorized by law," has apparently
convinced petitioner Beltran that the Constitution now requires the judge to personally examine the complainant
and his witnesses in his determination of probable cause for the issuance of warrants of arrest. This is not an
accurate interpretation.
What the Constitution underscores is the exclusive and personal responsibility of the issuing judge to satisfy himself
of the existence of probable cause. In satisfying himself of the existence of probable cause for the issuance of a
warrant of arrest, the judge is not required to personally examine the complainant and his witnesses. Following
established doctrine and procedure, he shall: (1) personally evaluate the report and the supporting documents
submitted by the fiscal regarding the existence of probable cause and, on the basis thereof, issue a warrant of
arrest; or (2) if on the basis thereof he finds no probable cause, he may disregard the fiscal's report and require the
submission of supporting affidavits of witnesses to aid him in arriving at a conclusion as to the existence of
probable cause.
Sound policy dictates this procedure, otherwise judges would be unduly laden with the preliminary examination and
investigation of criminal complaints instead of concentrating on hearing and deciding cases filed before their courts.
It has not been shown that respondent judge has deviated from the prescribed procedure. Thus, with regard to the
issuance of the warrants of arrest, a finding of grave abuse of discretion amounting to lack or excess of jurisdiction
cannot be sustained. The petitions fail to establish that public respondents, through their separate acts, gravely
abused their discretion as to amount to lack of jurisdiction. Hence, the writs of certiorari and prohibition prayed for
cannot issue.
WHEREFORE, finding no grave abuse of discretion amounting to excess or lack of jurisdiction on the part of the
public respondents, the Court Resolved to DISMISS the petitions in G. R. Nos. 82585, 82827 and 83979. The
Order to maintain the status quo contained in the Resolution of the Court en banc dated April 7, 1988 and reiterated
in the Resolution dated April 26, 1988 is LIFTED.

PENDON VS COURT OF APPEALS, November 16, 1990


Pre-typed questions as a basis of probable cause not valid. No searching questions

PEOPLE VS INTING, July 25, 1990


On 6 February 1988, Mrs. Editha Barba filed a letter-complaint against OIC-Mayor Dominador S. Regalado Jr. of
Tanjay, Negros Oriental with the Commission on Elections (COMELEC), for allegedly transferring her, a permanent
Nursing Attendant, Grade I, in the office of the Municipal Mayor to a very remote barangay and without obtaining
prior permission or clearance from COMELEC as required by law. Acting on the complaint, COMELEC directed
Atty. Gerardo Lituanas, Provincial Election Supervision of Dumaguete City: (1) to conduct the preliminary
investigation of the case; (2) to prepare and file the necessary information in court; (3) to handle the prosecution if
the evidence submitted shows a prima facie case and (3) to issue a resolution of prosecution or dismissal as the
case may be. After a preliminary investigation of Barba's complaint, Atty. Lituanas found a prima facie case. Hence,
on 26 September 1988, he filed with the Regional Trial Court (Branch 38. Dumaguete City) a criminal case for
violation of section 261, Paragraph (h), Omnibus Election Code against the OIC-Mayor. In an Order dated 30
September 1988, the court issued a warrant of arrest against the OIC Mayor. It also fixed the bail at P5,000.00 as
recommended by the Provincial Election Supervisor. However, in an order dated 3 October 1988 and before the
accused could be arrested, the trial court set aside its 30 September 1988 order on the ground that Atty. Lituanas is
not authorized to determine probable cause pursuant to Section 2, Article III of the 1987 Constitution. The court
stated that it "will give due course to the information filed in this case if the same has the written approval of the
Provincial Fiscal after which the prosecution of the case shall be under the supervision and control of the latter." In
another order dated 22 November 1988, the court gave Atty. Lituanas 15 days from receipt to file another
information charging the same offense with the written approval of the Provincial Fiscal. Atty. Lituanas failed to
comply with the order. Hence, in an order dated 8 December 1988, the trial court quashed the information. A
motion for reconsideration was denied. Hence, the petition.
ISSUE: Whether the approval of the Provincial Fiscal is necessary before the information filed by the Provincial
Election Supervisor may be given due course by the trial court.
HELD: As to the constitutional mandate that "xx no search warrant or warrant of arrest shall issue except upon
probable cause to be determined personally by the judge xx," (Article III, Section 2, Constitution) the determination
of probable cause is a function of the Judge. It is not for the Provincial Fiscal or Prosecutor nor for the Election
Supervisor to ascertain. Only the Judge and the Judge alone makes this determination. On the other hand, the
preliminary inquiry made by a Prosecutor does not bind the Judge. It merely assists him to make the determination
of probable cause. The Judge does not have to follow what the Prosecutor presents to him. By itself, the
Prosecutor's certification of probable cause is ineffectual. It is the report, the affidavits, the transcripts of

stenographic notes (if any), and all other supporting documents behind the Prosecutor's certification which are
material in assisting the Judge to make his determination. Thus, Judges and Prosecutors alike should distinguish
the preliminary inquiry which determines probable cause for the issuance of a warrant of arrest from the preliminary
investigation proper which ascertains whether the offender should be held for trial or released. Even if the two
inquiries are conducted in the course of one and the same proceeding, there should be no confusion about the
objectives. The determination of probable cause for the warrant of arrest is made by the Judge. The preliminary
investigation proper - whether or not there is reasonable ground to believe that the accused is guilty of the offense
charged and, therefore, whether or not be should be subjected to the expense, rigors and embarrassment of trial
is the function of the Prosecutor.
Preliminary investigation should be distinguished as to whether it is an investigation for the determination of a
sufficient ground for the filing of the information or it is an investigation for the determination of a probable cause for
the issuance of a warrant of arrest. The first kind of preliminary investigation is executive in nature. It is part of the
prosecution's job. The second kind of preliminary investigation which is more properly called preliminary
examination is judicial in nature and is lodged with the judge. The 1987 Constitution (Article IX C, Section 2)
mandates the COMELEC not only to investigate but also to prosecute cases of violation of election laws. This
means that the COMELEC is empowered to conduct preliminary investigations in cases involving election offenses
for the purpose of helping the Judge determine probable cause and for filing an information in court. This power is
exclusive with COMELEC. It is only after a preliminary examination conducted by the COMELEC through its
officials or its deputies that section 2, Article III of the 1987 Constitution comes in. This is so, because, when the
application for a warrant of arrest is made and the information is filed with the court, the judge will then determine
whether or not a probable cause exists for the issuance of a warrant of arrest. The trial court misconstrued the
constitutional provision when it quashed the information filed by the Provincial Election Supervisor. The order to get
the approval of the Provincial Fiscal is not only superfluous but unwarranted.

UMIL VS RAMOS, July 9, 1990


On 1 February 1988, the Regional Intelligence Operations Unit of the Capital Command (RIOU-CAPCOM) received
confidential information about a member of the NPA Sparrow Unit (liquidation squad) being treated for a gunshot
wound at the St. Agnes Hospital in Roosevelt Avenue, Quezon City. Upon verification, it was found that the
wounded person, who was listed in the hospital records as Ronnie Javelon, is actually Rolando Dural, a member of
the NPA liquidation squad, responsible for the killing of 2 CAPCOM soldiers the day before, or on 31 January 1988,
in Macanining Street, Bagong Barrio, Caloocan City. In view of this verification, Dural was transferred to the
Regional Medical Services of the CAPCOM, for security reasons. While confined thereat, or on 4 February 1988,
Dural was positively identified by eyewitnesses as the gunman who went on top of the hood of the CAPCOM
mobile patrol car, and fired at the 2 CAPCOM soldiers seated inside the car identified as T/Sgt. Carlos Pabon and
CIC Renato Manligot. As a consequence of this positive identification, Dural was referred to the Caloocan City
Fiscal who conducted an inquest and thereafter filed with the Regional Trial Court of Caloocan City an information
charging Rolando Dural alias Ronnie Javelon with the crime of "Double Murder with Assault Upon Agents of
Persons in Authority." (Criminal Case C-30112; no bail recommended). On 15 February 1988, the information was
amended to include, as defendant, Bernardo Itucal, Jr. who, at the filing of the original information, was still
unidentified. Meanwhile, on 6 February 1988, a petition for habeas corpus was filed with the Supreme Court on
behalf of Roberto Umil, Rolando Dural, and Renato Villanueva. The Court issued the writ of habeas corpus on 9
February 1988 and Fidel V. Ramos, Maj. Gen. Renato de Villa, Brig. Gen. Ramon Montano, and Brig. Gen.
Alexander Aguirre filed a Return of the Writ on 12 February 1988. Thereafter, the parties were heard on 15
February 1988. On 26 February 1988, however, Umil and Villanueva posted bail before the Regional Trial Court of
Pasay City where charges for violation of the Anti-Subversion Act had been filed against them, and they were
accordingly released.
ISSUE: Whether Dural can be validly arrested without any warrant of arrest for the crime of rebellion.
HELD: Dural, it clearly appears that he was not arrested while in the act of shooting the 2 CAPCOM soldiers nor
was he arrested just after the commission of the said offense for his arrest came a day after the said shooting
incident. Seemingly, his arrest without warrant is unjustified. However, Dural was arrested for being a member of
the New Peoples Army (NPA), an outlawed subversive organization. Subversion being a continuing offense, the
arrest of Rolando Dural without warrant is justified as it can be said that he was committing an offense when
arrested. The crimes of rebellion, subversion, conspiracy or proposal to commit such crimes, and crimes or
offenses committed in furtherance thereof or in connection therewith constitute direct assaults against the State and
are in the nature of continuing crimes. The arrest of persons involved in the rebellion whether as its fighting armed
elements, or for committing non-violent acts but in furtherance of the rebellion, is more an act of capturing them in
the course of an armed conflict, to quell the rebellion, than for the purpose of immediately prosecuting them in court
for a statutory offense. The arrest, therefore, need not follow the usual procedure in the prosecution of offenses
which requires the determination by a judge of the existence of probable cause before the issuance of a judicial

warrant of arrest and the granting of bail if the offense is bailable. Obviously, the absence of a judicial warrant is no
legal impediment to arresting or capturing persons committing overt acts of violence against government forces, or
any other milder acts but equally in pursuance of the rebellious movement. The arrest or capture is thus impelled by
the exigencies of the situation that involves the very survival of society and its government and duly constituted
authorities.

PADERANGA VS DRILON, April 19, 1990


On 16 October 1986, an information for multiple murder was filed in the Regional Trial Court, Gingoog City, against
Felipe Galarion, Manuel Sabit, Cesar Sabit, Julito Ampo, Eddie Torion, John Doe, Peter Doe and Richard Doe, for
the deaths on 1 May 1984 of Renato Bucag, his wife Melchora Bucag, and their son Renato Bucag II. Venue was,
however, transferred to Cagayan de Oro City per Administrative Matter 87-2-244. Only Felipe Galarion was tried
and found guilty as charged. The rest of the accused remained at large. Felipe Galarion, however, escaped from
detention and has not been apprehended since then. In an amended information filed on 6 October 1988, Felizardo
Roxas, alias "Ely Roxas," "Fely Roxas" and "Lolong Roxas," was included as a co-accused. Roxas retained Atty.
Miguel P. Paderanga as his counsel. As counsel for Roxas, Paderanga filed, among others, an Omnibus Motion to
dismiss, to Quash the Warrant of Arrest and to Nullify the Arraignment on 14 October 1988. The trial court in an
order dated 9 January 1989, denied the omnibus motion but directed the City Prosecutor "to conduct another
preliminary investigation or reinvestigation in order to grant the accused all the opportunity to adduce whatever
evidence he has in support of his defense." In the course of the preliminary investigation, through a signed affidavit,
Felizardo Roxas implicated Atty. Paderanga in the commission of the crime charged. The City Prosecutor of
Cagayan de Oro City inhibited himself from further conducting the preliminary investigation against Paderanga at
the instance of the latter's counsel, per his resolution dated 7 July 1989. In his first indorsement to the Department
of Justice, dated 24 July 1989, said city prosecutor requested the Department of Justice to designate a state
prosecutor to continue the preliminary investigation against Paderanga. In a resolution dated 6 September 1989,
the State Prosecutor Henrick F. Gingoyon, who was designated to continue with the conduct of the preliminary
investigation against Paderanga, directed the amendment of the previously amended information to include and
implead Paderanga as one of the accused therein. Paderanga moved for reconsideration, contending that the
preliminary investigation was not yet completed when said resolution was promulgated, and that he was deprived of
his right to present a corresponding counter-affidavit and additional evidence crucial to the determination of his
alleged "linkage" to the crime charged. The motion was, however, denied by Gingoyon in his order dated 29
January 1990. From the aforesaid resolution and order, Paderanga filed a Petition for Review with the Department
of Justice. Thereafter, he submitted a Supplemental Petition with Memorandum, and then a Supplemental
Memorandum with Additional Exculpatory/Exonerating Evidence Annexed, attaching thereto an affidavit of Roxas
dated 20 June 1990 and purporting to be a retraction of his affidavit of 30 March 1990 wherein he implicated
Paderanga. On 10 August 1990, the Department of Justice, through Undersecretary Silvestre H. Bello III, issued
Resolution 648 dismissing the said petition for review. His motion for reconsideration having been likewise denied,
Paderanga then filed the petition for mandamus and prohibition before the Supreme Court.
ISSUE: Whether there is no prima facie evidence, or probable cause, or sufficient justification to hold Paderanga to
a tedious and prolonged public trial.
HELD: A preliminary investigation is defined as an inquiry or proceeding for the purpose of determining whether
there is sufficient ground to engender a well-founded belief that a crime cognizable by the Regional Trial Court has
been committed and that the respondent is probably guilty thereof, and should be held for trial. The quantum of
evidence now required in preliminary investigation is such evidence sufficient to "engender a well-founded belief"
as to the fact of the commission of a crime and the respondent's probable guilt thereof. A preliminary investigation
is not the occasion for the full and exhaustive display of the parties' evidence; it is for the presentation of such
evidence only as may engender a well-grounded belief that an offense has been committed and that the accused is
probably guilty thereof. Preliminary investigation is generally inquisitorial, and it is often the only means of
discovering the persons who may be reasonably charged with a crime, to enable the fiscal to prepare his complaint
or information. It is not a trial of the case on the merits and has no purpose except that of determining whether a
crime has been committed and whether there is probable cause to believe that the accused is guilty thereof, and it
does not place the person against whom it is taken in jeopardy. The institution of a criminal action depends upon
the sound discretion of the fiscal. He has the quasi-judicial discretion to determine whether or not a criminal case
should be filed in court. Hence, the general rule is that an injunction will not be granted to restrain a criminal
prosecution. The case of Brocka, et al. vs. Enrile, et al. cites several exceptions to the rule, to wit: (a) To afford
adequate protection to the constitutional rights of the accused; (b) When necessary for the orderly administration of
justice or to avoid oppression or multiplicity of actions; (c) When there is a prejudicial question which is sub-judice;
(d) When the acts of the officer are without or in excess of authority; (e) Where the prosecution is under an invalid
law, ordinance or regulation; (f) When double jeopardy is clearly apparent; (g) Where the court has no jurisdiction
over the offense; (h) Where it is a case of persecution rather than prosecution; (i) Where the charges are manifestly

false and motivated by the lust for vengeance; and (j) When there is clearly no prima facie case against the
accused and a motion to quash on that ground has been denied. A careful analysis of the circumstances obtaining
in the present case, however, will readily show that the same does not fall under any of the aforesaid exceptions.

DEPARTMENT OF HEALTH VS SY CHI SIONG, GR 85289

PEOPLE VS VILLANUEVA, 110 SCRA 465


The judge may validly refuse to issue warrants of arrest if he believes that there is no probable cause to for their
issuance; despite the findings of probable cause by the filing Prosecutor since that was for purposes of filing only.

PLACER VS VILLANUEVA, 126 SCRA 463

TOLENTINO VS VILLALUZ, JULY 27,1987

CRUZ VS GATAN, 74 SCRA 226

OLAES VS PEOPLE, 155 SCRA 486


Adolfo Olaes and Linda M. Cruz were charged for violation of the Dangerous Drugs Act. Olaes and Cruz filed a
petition for certiorari and prohibition with preliminary injunction, challenging the admission by Judge Alicia L. Santos
(in her capacity as Presiding Judge of the Regional Trial Court of Olongapo City, Branch 73) of evidence seized by
virtue of an allegedly invalid search warrant and of an extrajudicial confession taken from them without according
them the right to assistance of counsel; and thus seek to restrain further proceedings in the criminal case against
them and ask that they be acquitted with the setting aside of the questioned orders (the facts do not provide the
disposition of the said orders). Olaes and Cruz claim that the search warrant issued by the judge is unconstitutional
because it does not indicate the specific offense they are supposed to have committed. There is, therefore,
according to them, no valid finding of probable cause as a justification for the issuance of the said warrant in
conformity with the Bill of Rights.
ISSUE: Whether the lack of specific section of the Dangerous Drugs Act renders the caption vague, and negate the
claim that the specific offense was committed to serve as basis for the finding of probable cause.
HELD: No. The search warrant issued does not come under the strictures of the Stonehill doctrine. While in the
case cited, there was a bare reference to the laws in general, without any specification of the particular sections
thereof that were alleged to have been violated out of the hundreds of prohibitions contained in such codifications,
there is no similar ambiguity herein. While it is true that the caption of the search warrant states that it is in
connection with "Violation of RA 6425, otherwise known as the Dangerous Drugs Acts of 1972," it is clearly recited
in the text thereof that "There is probable cause to believe that Adolfo Olaes alias 'Debie' and alias 'Baby' of No.
628 Comia St., Filtration, Sta. Rita, Olongapo City, has in their possession and control and custody of marijuana
dried stalks/leaves/seeds/cigarettes and other regulated/prohibited and exempt narcotics preparations which is the
subject of the offense stated above." Although the specific section of the Dangerous Drugs Act is not pinpointed,
there is no question at all of the specific offense alleged to have been committed as a basis for the finding of
probable cause. The search warrant also satisfies the requirement in the Bill of Rights of the particularity of the
description to be made of the "place to be searched and the persons or things to be seized."

GERONIMO VS RAMOS, 136 SCRA 435


Warrants of arrest issued in 70 plus cases against several accused after less than 1 hour from receipt of the
records of the cases are not valid. Judge could not have determined probable cause in so short a time

ENRILE VS SALAZAR, ET AL., G.R.NO. 92163


Warrant of arrest issued against Senator Enrile after1 hour and 20 minutes from receipt of the records of the case
consisting of several thousands of pages is valid. There is no need to read all the affidavits or evidence therein. It is
sufficient that he is convinced of the existence of probable cause.
On February 27, 1990, Senator Juan Ponce Enrile was arrested by law enforcement officers led by NBI Director
Alfredo Lim on the strength of a warrant of arrest issued by the respondent judge, HON. JAIME SALAZAR,

Regional trial Court, Branch 103, Quezon City in Criminal Case No. 90-10941. The warrant was issued on an
information signed and filed earlier in the day by Senior State Prosecutor AURELIO TRAMPE charging Senator
Enrile, the spouses Rebecco and Erlinda Panlilio, and Gregorio Honasan with the crime of rebellion with murder
and multiple frustrated murder allegedly committed during the period of the failed coup attempt from November 29
to December 10, 1990. Senator Enrile was taken to and held overnight at the NBI Headquarters on Taft Ave.,
Manila, WITHOUT BAIL, NONE HAVING BEEN RECOMMENDED IN THE INFORMATION AND NONE FIXED IN
THE WARRANT OF ARREST.
On February 28, 1990, petitioner through counsel filed a petition for Habeas Corpus alleging that he was deprived
of his constitutional rights in being, or having been:
a. held to answer for a criminal offense which does not exist in the statute books;
b. charged with a criminal offense in an information for which no complaint was initially filed or preliminary
investigation was conducted, hence, he was denied due process;
c. denied the right to bail; and
d. arrested or detained on the strength of warrant issued without the judge who issued it first having
personally determined the existence of probable cause.
HELD:
The parties oral and written arguments presented the following options:
1. Abandon the Hernandez Doctrine and adopt the dissenting opinion of Justice Montemayor that rebellion
cannot absorb more serious crimes; On the first option, 11 justices voted AGAINST abandoning
Hernandez. Two members felt that the doctrine should be re-examined. In view of the majority, THE
RULING REMAINS GOOD LAW, ITS SUBSTANTIVE AND LEGAL BASES HAVE WITHSTOOD ALL
SUBSEQUENT CHALLENGES AND NO NEW ONES ARE PRESENTED HERE PERSUASIVE ENOUGH
TO WARRANT A COMPLETE REVERSAL. This is so because of the fact that the incumbent President
(exercising legislative powers under the 1986 Freedom Constitution) repealed PD No. 942 which added a
new provision of the Revised Penal Code, particularly Art. 142-A which sought to nullify if not repealed the
Hernandez Doctrine. In thus acting, the President in effect by legislative fiat reinstated the Hernandez as a
binding doctrine with the effect of law. The Court can do no less than accord it the same recognition,
absent any sufficiently powerful reason against so doing.
2. Hold Hernandez Doctrine applicable only to offenses committed in furtherance, or as necessary means for
the commission, of rebellion, BUT NOT TO ACTS COMMITTED IN THE COURSE OF A REBELLION
WHICH ALSO CONSTITUTE COMMON CRIMES OF GRAVE OR LESS GRAVE CHARACTER; On the
second option, the Supreme Court was unanimous in voting to reject the same though four justices believe
that the arguments in support thereof is not entirely devoid of merit.
3. Maintain Hernandez Doctrine as applying to make rebellion absorb all other offenses committed in its
course, whether or not necessary to its commission or in furtherance thereof. With the rejection of the first
two options, the Hernandez Doctrine remains a binding doctrine operating to prohibit the complexing of
rebellion with any other offense committed on the occasion thereof, either as a means necessary to its
commission or as unintended effect of an activity that constitutes rebellion.
On the issues raised by the petitioner:
a. By a vote of 11-3, the Court ruled that the information filed against the petitioner does in
fact charge an offense despite the objectionable phrasing that would complex rebellion
with murder and multiple frustrated murder, that indictment is to be read as charging
SIMPLE REBELLION. The petitioners contention that he was charged with a crime that
does not exist in the statute books, WHILE TECHNICALLY CORRECT SO FAR AS THE
COURT RULED THAT REBELLION MAY NOT BE COMPLEXED WITH OTHER
OFFENSES COMMITTED ON THE OCCASION THEREOF, MUST THEREFORE BE
DISMISSED AS A MERE FLIGHT OF RHETORIC. Read in the context of Hernandez, the
information does indeed charge the petitioner with a crime defined and punished by the
Revised Penal Code: SIMPLE REBELLION.
b. Was the petitioner charged without a complaint having been initially filed and/or preliminary
investigation conducted? The record shows that a complaint for simple rebellion against
petitioner was filed by the NBI Director and that based on the strength of said complaint a
preliminary investigation was conducted by the respondent prosecutors culminating in the
filing of the questioned information. THERE IS NOTHING INHERENTLY IRREGULAR OR
CONTRARY TO LAW IN FILING AGAINST A RESPONDENT AN INDUCTMENT FOR AN
OFFENSE DIFFERENT FROM WHAT IS CHARGED IN THE INITIATORY COMPLAINT,

IF WARRANTED BY THE EVIDENCE DEVELOPED DURING THE PRELIMINARY


INVESTIGATION.
c. The petitioner claims that the warrant issued is void because it was issued barely one hour
and twenty minutes after the case was raffled to the respondent judge which could hardly
gave him sufficient time to personally go over the voluminous records of the preliminary
investigation. Also, the petitioner claims that the respondent judge issued the warrant for
his arrest without first personally determining the existence of probable cause by
examining under oath or affirmation the complainant and his witnesses, in violation of Art.
III, Section 2, of the Constitution. This Court has already ruled that it is not unavoidable
duty of the judge to make such a personal examination, it being sufficient that he follows
established procedure by PERSONALLY EVALUATING THE REPORT AND THE
SUPPORTING DOCUMENT SUBMITTED BY THE PROSECUTOR. MEREBY BECAUSE
SAID RESPONDENT JUDGE HAD WHAT SOME MIGHT CONSIDER ONLY A
RELATIVELY BRIEF PERIOD WITHIN WHICH TO COMPLY WITH THAT DUTY , GIVES
NO REASON TO ASSUME THAT HE HAD NOT, OR COULD NOT HAVE, SO
COMPLIED; NOR DOES THAT SINGLE CIRCUMSTANCE SUFFICE TO OVERCOME
THE LEGAL PRESUMPTION THAT OFFICIAL DUTY HAS BEEN REGULARLY
PERFORMED.
d. Petitioner also claims that he is denied of his constitutional right to bail. In the light of the
Courts affirmation of Hernandez as applicable to petitioners case, and of the logical and
necessary corollary that the information against him should be considered as charging only
the crime of simple rebellion which is bailable before conviction, THAT MUST NOW BE
ACCEPTED AS A CORRECT PROPOSITION.

ADDITIONAL NOTES:
Was a petition for Habeas Corpus before the Supreme Court the appropriate vehicle for asserting a
right to bail or vindicating its denial?
The Supreme Court held that the criminal case before the respondent judge is the normal venue for
invoking the petitioners right to have provisional liberty pending trial and judgment. The correct
course was for the petitioner to invoke that jurisdiction by filing a petition to be admitted to bail,
claiming a right to bail per se or by reason of the weakness of the evidence against him. ONLY
AFTER THAT REMEDY WAS DENIED BY THE TRIAL COURT SHOULD THE REVIEW
JURISDICTION OF THE SUPREME COURT BE INVOKED, AND EVEN THEN, NOT WITHOUT
FIRST APPLYING TO THE COURT OF APPEALS IF APPROPRIATE RELIEF WAS ALSO
AVAILABLE THERE.
Even if we assume that the petitioners premise that the information charges a non-existent crime,
it would not excuse or justify his improper choice of remedies. Under either hypothesis, the obvious
recourse would have been a motion to quash brought in the criminal action before the respondent
judge.

WARRANTLESS SEARCHES AND SEIZURES--WHEN VALID OR NOT; WHETHER "OPERATION KAPKAP" IS


VALID; WARRANTLESS SEARCH OF ALLEGED OBSCENE MAGAZINES
PEOPLE VS MENGOTE, G.R. No. 87059
Arrest without warrant because of a bulging tummy which looks like a gun tucked therein is not valid. There was
neither personal knowledge nor probable cause.
The Western Police District received a telephone call from an informer that there were three suspicious looking
persons at the corner of Juan Luna and North Bay Boulevard in Tondo, Manila. A surveillance team of
plainclothesmen was forthwith dispatched to the place. The patrolmen saw two men looking from side to side, one
of whom holding his abdomen. They approached the persons and identified themselves as policemen, whereupon
the two tried to run but unable to escape because the other lawmen surrounded them. The suspects were then
searched. One of them the accused-appellant was found with a .38 caliber with live ammunitions in it, while his
companion had a fan knife. The weapons were taken from them and they were turned over to the police
headquarters for investigation. An information was filed before the RTC convicting the accused of illegal possession
of firearm arm. A witness testified that the weapon was among the articles stolen at his shop, which he reported to
the police including the revolver. For his part, Mengote made no effort to prove that he owned the fire arm or that he
was licensed to possess it but instead, he claimed that the weapon was planted on him at the time of his arrest. He
was convicted for violation of P.D.1866 and was sentenced to reclusion perpetua. In his appeal he pleads that the
weapon was not admissible as evidence against him because it had been illegally seized and therefore the fruit of a
poisonous tree.
ISSUE: Whether or not the warrantless search and arrest was illegal.
HELD: An evidence obtained as a result of an illegal search and seizure inadmissible in any proceeding for any
purpose as provided by Art. III sec 32 of the Constitution. Rule 113 sec.5 of the Rules of Court, provides arrest
without warrant lawful when: (a) the person to be arrested has committed, is actually committing, or is attempting to
commit an offense, (b) when the offense in fact has just been committed, and he has personal knowledge of the
facts indicating the person arrested has committed it and (c) the person to be arrested has escaped from a penal
establishment or a place where he is serving final judgment or temporarily confined while his case is pending, or
has escaped while being transferred from one confinement to another.
These requirements have not been established in the case at bar. At the time of the arrest in question, the accused
appellant was merely looking from side to side and holding his abdomen, according to the arresting officers
themselves. There was apparently no offense that has just been committed or was being actually committed or at
least being attempt by Mengote in their presence. Moreover a person may not be stopped and frisked in a broad
daylight or on a busy street on unexplained suspicion.
Judgment is reversed and set aside. Accused-appellant is acquitted.

PEOPLE VS GO, 354 SCRA 338


Search of a gun which could be seen tucked in the waist of the accused in a nightclub is valid. Likewise, the
sachets of shabu seen on the front seat of the car of the accused when he opened the car are admissible under the
plain view doctrine.
Where the gun tucked in a persons waist is plainly visible to the police, no search warrant is necessary and in the
absence of any license for said firearm, he may be arrested at once as he is in effect committing a crime in the
presence of the police officers. No warrant is necessary in such a situation, it being one of the recognized
exceptions under the Rules.
As a consequence of the accuseds valid warrantless arrest inside the nightclub, he may be lawfully searched for
dangerous weapons or anything which may be used as proof of the commission of an offense, without a search
warrant in accordance with Section 12, Rule 126. This is a valid search incidental to a lawful arrest.
In fact, the subsequent discovery in his car which was parked in a distant place from where the illegal possession of
firearm was committed [after he requested that he will bring his car to the Police Station after his warrantless arrest)
, of a drug paraphernalia and shabu, CANNOT BE SAID TO HAVE BEEN MADE DURING AN ILLEGAL SEARCH.
As such, the items do not fall under the exclusionary rule and the unlicensed firearms, drug paraphernalia and the
shabu, can be used as evidence against the accused.

PITA VS CA, 178 SCRA 362


Requisites before the Mayor could confiscate magazines which are allegedly obscene.
On December 1 and 3, 1983, pursuing an Anti-Smut Campaign initiated by the Mayor of the City of Manila, Ramon
D. Bagatsing, elements of the Special Anti-Narcotics Group, Auxilliary Services Bureau, Western Police District,
INP of the Metropolitan Police Force of Manila, seized and confiscated from dealers, distributors, newsstand

owners and peddlers along Manila sidewalks, magazines, publications and other reading materials believed to be
obscene, pornographic and indecent and later burned the seized materials in public at the University belt along
C.M. Recto Avenue, Manila, in the presence of Mayor Bagatsing and several officers and members of various
student organizations. Among the publications seized, and later burned, was "Pinoy Playboy" magazines published
and co-edited by plaintiff Leo Pita.
Plaintiff filed a case for injunction with prayer for issuance of the writ of preliminary injunction against Mayor
Bagatsing and Narcisco Cabrera, as superintendent of Western Police District of the City of Manila, seeking to
enjoin said defendants and their agents from confiscating plaintiffs magazines or from preventing the sale or
circulation thereof claiming that the magazine is a decent, artistic and educational magazine which is not per se
obscene, and that the publication is protected by the Constitutional guarantees of freedom of speech and of the
press. Plaintiff also filed an Urgent Motion for issuance of a temporary restraining order against indiscriminate
seizure, confiscation and burning of plaintiff's "Pinoy Playboy" Magazines, pending hearing on the petition for
preliminary injunction. The Court granted the temporary restraining order. The case was set for trial upon the lapse
of the TRO. RTC ruled that the seizure was valid. This was affirmed by the CA.
ISSUE: Whether or Not the seizure violative of the freedom of expression of the petitioner.
HELD: Freedom of the press is not without restraint as the state has the right to protect society from pornographic
literature that is offensive to public morals, as indeed we have laws punishing the author, publishers and sellers of
obscene publications. However, It is easier said than done to say, that if the pictures here in question were used
not exactly for art's sake but rather for commercial purposes, the pictures are not entitled to any constitutional
protection. Using the Kottinger rule: the test of obscenity is "whether the tendency of the matter charged as
obscene, is to deprave or corrupt those whose minds are open to such immoral influences and into whose hands a
publication or other article charged as being obscene may fall." Another is whether it shocks the ordinary and
common sense of men as an indecency. Ultimately "whether a picture is obscene or indecent must depend upon
the circumstances of the case and that the question is to be decided by the "judgment of the aggregate sense of
the community reached by it." The government authorities in the instant case have not shown the required proof to
justify a ban and to warrant confiscation of the literature First of all, they were not possessed of a lawful court order:
(1) finding the said materials to be pornography, and (2) authorizing them to carry out a search and seizure, by way
of a search warrant. The court provides that the authorities must apply for the issuance of a search warrant from a
judge, if in their opinion an obscenity seizure is in order and that;
1. The authorities must convince the court that the materials sought to be seized are obscene and pose a
clear and present danger of an evil substantive enough to warrant State interference and action;
2. The judge must determine whether or not the same are indeed obscene. The question is to be resolved on
a case-to-case basis and on the judges sound discretion;

WARRANTLESS SEARCH AND SEIZURE BY A PRIVATE PERSON


PEOPLE VS MENDOZA, 301 SCRA 66

SILAHIS INTERNATIONAL VS SOLUTA, 482 SCRA 660


The petitioner suspects that the respondents who are officers of the Silahis International Hotel Union were using
the Union Office located inside the hotel in the sale or use of marijuana, dollar smuggling, and prostitution. They
arrived at the said conclusion through surveillance.
In the morning of January 11, 1988, while the respondent union officer was opening the Union Office, security
officers of the plaintiff entered the union office despite objections thereto by forcibly opening the same. Once inside
the union office they started to make searches which resulted in the confiscation of a plastic bag of marijuana. An
information for violation of the dangerous drugs act was filed against the respondent before the RTC of Manila
which acquitted them on the ground that the search conducted was illegal since it was warrantless and without
consent by the respondents.
After their acquittal, the respondents filed a case for Malicious Prosecution against the petitioner for violation of Art.
32 of the Civil Code. After trial, the Regional Trial Court held that petitioners are liable for damages as a result of
an illegal search. The same was affirmed by the Court of Appeals.
ISSUE: Whether the warrantless search conducted by the petitioners (private individual and corporation) on the
union office of the private respondents is valid.
HELD: The search is not valid and they are civilly liable under Art. 32 of the Civil Code. The fact that the union
office is part of the hotel owned by the petitioners does not justify the warrantless search. The alleged reports that

the said union office is being used by the union officers for illegal activities does not justify their acts of barging into
the said office without the consent of the union officers and without a search warrant. If indeed there was
surveillance made, then they should have applied for a search warrant.
NOTE: The ruling in People vs. Andre Marti is not applicable here because in Marti, a criminal case, the
issue was whether an act of a private individual, allegedly in violation of ones constitutional rights may be
invoked against the State. In other words, the issue in Marti is whether the evidence obtained by a private
person acting in his private capacity without the participation of the State, is admissible.

PEOPLE VS ANDRE MARTI, G.R. 81561


Andre Marti and his wife went to Manila Packing and Export Forwarders, carrying with them four gift wrapped
packages to be delivered to his friend in Zurich, Switzerland. Anita Reyes (wife of the proprietor) asked if she could
inspect the packages, however, Marti refused assuring that it only contained books, cigars and gloves as gift to his
friend.
Before delivery to Bureau of Customs/Posts, the proprietor Job Reyes, following standard operating procedure,
opened the boxes for final inspection. When he opened Marti's boxes, a particular odor emitted therefrom and he
soon found out that the boxes contained dried marijuana leaves. He reported the incident to the NBI who
acknowledged custody of the incident. Marti was convicted for violation of R.A. 6425, otherwise known as the
Dangerous Drugs Act.
ISSUE: Marti contends that the evidence had been obtained in violation of his constitutional rights against
unreasonable search and seizure and privacy of communication.
HELD: Evidence sought to be excluded was primarily discovered and obtained by a private person, acting in a
private capacity and without the intervention and participation of State authorities. In the absence of governmental
interference, the liberties guaranteed by the Constitution cannot be invoked against the State. Mere presence of
NBI agents does not convert it to warrantless search and siezure. Merely to look at that which is plain sight is not
search. Having observed that which is open, where no trespass has been committed is not search.

VALID WARRANTLESS SEARCH AND SEIZURE:


1. SEARCHES MADE INCIDENTAL TO A VALID ARREST
PEOPLE VS SANGKI ARA, December 23, 2009

PEOPLE VS PEAFLORA, GR 175604

MORENO VS AGO CHI, 12 PHIL 439

PEOPLE VS ANG CHUN KIT, 251 SCRA 660

PEOPLE VS LUA, 256 SCRA 539

PEOPLE VS FIGUEROA, 248 SCRA 679

NOLASCO VS PANO, 139 SCRA 541


One who was arrested on board a passenger jeepney may not be brought to her residence and then search the
same. That could not be considered as search incidental to a valid arrest.
Milagros Aguilar-Roque was arrested together with Cynthia Nolasco by the Constabulary Security Group (CSG).
Milagros had been wanted as a high ranking officer of the CPP. The arrest took place at 11:30 a.m. of August 6,
1984. At noon of the same day, her premises were searched and 428 documents, a portable typewriter and 2
boxes were seized.
Earlier that day, Judge Cruz Pao issued a search warrant to be served at Aguilar-Roques leased residence
allegedly an underground house of the CPP/NPA. On the basis of the documents seized, charges of subversion

and rebellion by the CSG were filed by but the fiscals office merely charged her and Nolasco with illegal
possession of subversive materials. Aguilar-Roque asked for suppression of the evidence on the ground that it was
illegally obtained and that the search warrant is void because it is a general warrant since it does not sufficiently
describe with particularity the things subject of the search and seizure, and that probable cause has not been
properly established for lack of searching questions propounded to the applicants witness.
ISSUE: WON the search warrant was valid?
HELD: No, Section 3, Article IV of the Constitution, guarantees the right of the people to be secure in their persons,
houses, papers and effects against unreasonable searches and seizures of whatever nature and for any purpose. It
also specifically provides that no Search Warrant shall issue except upon probable cause to be determined by the
Judge or such other responsible officer as may be authorized by law, after examination under oath or affirmation of
the complainant and the witnesses he may produce, and particularly describing the place to be searched and the
things to be seized.
It is at once evident that the foregoing Search Warrant authorizes the seizure of personal properties vaguely
described and not particularized. It is an all- embracing description which includes everything conceivable regarding
the Communist Party of the Philippines and the National Democratic Front. It does not specify what the subversive
books and instructions are; what the manuals not otherwise available to the public contain to make them subversive
or to enable them to be used for the crime of rebellion. There is absent a definite guideline to the searching team as
to what items might be lawfully seized thus giving the officers of the law discretion regarding what articles they
should seize as, in fact, taken also were a portable typewriter and 2 wooden boxes.
It is thus in the nature of a general warrant and infringes on the constitutional mandate requiring particular
description of the things to be seized. In the recent rulings of this Court, search warrants of similar description were
considered null and void for being too general.

2. SEARCHES OF MOVING VEHICLES


PEOPLE VS BELEN MARIACOS, G.R. No. 18861, June 16, 2010
Tip that the accused has with her marijuana on her bag at the top load of a passenger jeepney justifies the police
authorities to conduct a valid Warrantless search since they have no more time to secure a search warrant.

ESPANO VS CA, 288 SCRA 588


If accused was arrested on the street in front of his house selling prohibited drugs, the arresting officers may not
search his house without warrant based on search incidental to a valid arrest rule.

CARROL VS US, 267 US 132


ISSUE: Search without a warrant of an automobile, and seizure therein of liquor subject to seizure and destruction
under the Prohibition Act, do not violate the Amendment, if made upon probable cause, i.e., upon a belief,
reasonably arising out of circumstances known to the officer, that the vehicle contains such contraband liquor.
HELD: The seizure is legal if the officer, in stopping and searching the vehicle, has reasonable or probable cause
for believing that contraband liquor is being illegally transported in it.
"When, in any prosecution commenced on account of the seizure of any vessel, goods, wares, or merchandise,
made by any collector or other officer, under any Act of Congress authorizing such seizure, judgment is rendered
for the claimant, but it appears to the court that there was reasonable cause of seizure, the court shall cause a
proper certificate thereof to be entered, and the claimant shall not, in such case, be entitled to costs, nor shall the
person who made the seizure, nor the prosecutor, be liable to suit or judgment on account of such suit or
prosecution: Provided, That the vessel, goods, wares, or merchandise be, after judgment, forthwith returned to
such claimant or his agent."

PEOPLE VS LO HO WING, 190 SCRA 122


In July 1987, the Special Operations Group of the CIS received a tip from one of its informers about an organized
group engaged in importation of illegal drugs and smuggling of contraband items. To infiltrate the crime syndicate,
they recruited confidential men and deep penetration agents under OPLAN SHARON 887. One such agent was
Reynaldo Tia (the discharged/accused). As an agent, he submitted regular reports of undercover activities of
suspected syndicates. CAPTAIN PALMERA, head of oplan sharon 887, in turned informed the Dangerous Drugs
Board of Tias activities.
Tia was introduced to his co-accused Lim Cheng Huat by another agent named George. Lim wanted a male
travelling companion for his business trips abroad. Tia offered his services and was hired by Lim. Later, Tia was

introduced to Peter Lo (alias of accused/appellant Lo Ho Wing), the later turning out to be Tias intended
companion.
Appellant Lo Ho Wing and Tia left for Hong Kong on October 4, 1987. Tia telephoned Capt. Palmera that they
would return to the Philippines on October 6. From Hong Kong, the two proceeded to Guangzhou in mainland
China. There, appeallant Lo Ho Wing bought six (6) cans of tea. Tia saw these 6 bags when they were opened for
examination. That evening, they went to Lo Ho Wings room and he saw two other men with him. One was fixing
the tea bags, while the other was burning a substance on a piece of aluminum foil using a lighter. Appellant Lo Ho
Wing joined the second man and sniffed the smoke emitted by the burning substance. When Tia asked Lo Ho Wing
what cargo they would bring to Manila, the latter replied that they would be bringing Chinese drugs.
The next day en route to Manila, customs examiners inspected the bags containing the tin cans of tea. Since the
bags were not closely examined, appellant Lo Ho Wing and Tia were cleared. In Manila, They were met by Lim
Cheng Huat. Appellant Lo Ho Wing and Tia boarded a taxi from the airport and loaded their luggage in the taxis
compartment. Lim Cheng Huat followed them in another taxi.
Meanwhile, a team composed by Capt. Palmera positioned themselves in strategic areas around the airport. The
CIS men who first saw Lo Ho and Tia followed them. Along Imelda Avenue, the CIS car overtook the taxi ridden by
Lo Ho Wing and Tia , forcing the taxi driver to stop his vehicle. The CIS team asked the taxi driver to open the
baggage compartment. The CIS team asked permission to search their luggage.
A tin can of tea was taken out of the compartment. Sgt. Cayabyab of the CIS pried the lid open and pressed it in
the middle to pull out the contents. Crystalline white powder resembling crushed alum came out. Suspecting the
crystalline powder to be a dangerous drug, he had the three travelling bags opened for inspection. All the bags
threshed out a total of six tin cans. Tia and appellant were taken to the CIS headquarters for questioning.
Meanwhile, the second taxi carrying Lim Cheng Huat sped in attempt to escape. However, they were later
captured.
Samples from the bag tested positive for methamphetamine. The three suspects were indicted for violating Art. III,
sec.15 of the Dangerous Drug Act. Appellant Lo Ho Wing and Lim Cheng Huat were sentenced to suffer life
imprisonment and to pay a fine of P25,000 each. Reynaldo Tia was discharged as a state witness. The trial court
gave full credence to the testimonies of government agents since the presumption of regularity in the performance
of official duties were in their favor.
ISSUES:
1. Was the warrantless search valid?
2. Are the effects taken admissible as evidence?
HELD:
1. This is a case of search on a moving vehicle which is one of the well-known exceptions to the valid
warrantless search and seizure. To stilol get a search warrant from a judge would allow the accused go
scot-free.
2. Since the search and seizure are valid, the evidence obtained is admissible as evidence in any proceeding.
3. Seizure of goods concealed to avoid duties/taxes (Valid)

MUSTANG LUMBER VS CA, 257 SCRA 430

3. CUSTOMS SEARCHES; SEIZURE OF GOODS CONCEALED TO AVOID DUTIES/TAXES


PAPA VS MAGO, 22 SCRA 857

PACIS VS PAMARAN, 56 SCRA 16

HIZON VS CA, 265 SCRA 517

PEOPLE VS QUE, 265 SCRA 721

4. SEIZURE OF EVIDENCE IN PLAIN VIEW


PEOPLE VS VALDEZ, 341 SCRA 25
The policemen were informed that the accused had fully grown marijuana planted in his yard. They went to see for
themselves the tip and found the same to be true. They arrested the accused and took pictures of him beside his
marijuana plants and thereafter uprooted said plant, again with his pictures. There was no valid Warrantless search
under the plain view doctrine since they were looking for it and not inadvertently come across the illegal items.

MICLAT JR VS PEOPLE, GR 176077

ELENITA FAJARDO VS PEOPLE, GR 190889


Plain view as a valid exception to a Warrantless searches and seizure
Under the plain view doctrine, objects falling in the plain view of an officer, who has a right to be in the position to
have that view, are subject to seizure and may be presented as evidence. It applies when the following requisites
concur: (a) the law enforcement officer in search of the evidence has a prior justification for an intrusion or is in a
position from which he can view a particular area; (b) the discovery of the evidence in plain view is inadvertent; and
(c) it is immediately apparent to the officer that the item he observes may be evidence of a crime, contraband, or
otherwise subject to seizure. The law enforcement officer must lawfully make an initial intrusion or properly be in a
position from which he can particularly view the area. In the course of such lawful intrusion, he came inadvertently
across a piece of evidence incriminating the accused. The object must be open to eye and hand, and its discovery
inadvertent.

HARRIS VS US, 390 US 234

PEOPLE VS DAMASO, 212 SCRA 547


In order that there is a valid waiver to a warrantless search, the waiver or consent should be given by the person
affected, not just anybody. Example: The landlady could not give a valid consent to the search of a room occupied
by a tenant. Said tenant himself should give the consent in order to be valid. The doctrine in Lopez vs.
Commissioner to the effect that it could be given by any occupant of a hotel room being rented by the respondent is
deemed abandoned.

PEOPLE VS VELOSO, 252 SCRA 135

PEOPLE VS LESANGIN, 252 SCRA 213

5. CONSENTED SEARCHES; WHEN THERE IS WAIVER OF RIGHT OR GIVES HIS CONSENT


PEOPLE VS BAULA, 344 SCRA 663
In case of consented searches or waiver of the constitutional guarantee against obtrusive searches, it is
fundamental that to constitute waiver, IT MUST APPEAR THAT THE RIGHT EXISTS; THE PERSONS INVOLVED
HAD KNOWLEDGE, EITHER ACTUAL OR CONSTRUCTIVE, of the existence of such right. The third condition did
not exist in the instant case. Neither was the search incidental to a valid warrantless arrest. (PEOPLE VS.
FIFUEROA, July 6, 2000) An alleged consent to a warrantless search and seizure cannot be based merely on the
presumption of regularity in the performance of official duty. THE PRESUMPTION BY ITSELF, CANNOT PREVAIL
AGAINST THE CONSTITUTIONALLY PROTECTED RIGHTS OF AN INDIVIDUAL, AND ZEAL IN THE PURSUIT
OF CRIMINALS CANNOT ENNOBLE THE USE OF ARBITRARY METHODS THAT THE CONSTITUTION ITSELF
ABHORS.

VEROY VS LAYAGUE, 210 SCRA 97

The consent given by the owner of the house to the soldiers to look for rebel soldiers does not justify them to
search for unlicensed firearm inside the house by opening cabinets therein.
If the owner of the house allowed the policemen to enter his house because they are searching for rebel soldiers
but when inside the house, they instead seized an unlicensed firearm, there is no consent to a warrantless search.
DE GARCIA VS LOCSIN, 65 PHIL 689

LOPEZ VS COMMISSIONER, 65 SCRA 336

PEOPLE VS DAMASO, 212 SCRA 547


Consent given by the owner of the apartment for the police to search the room being rented to Basilio Damaso is
not valid.
In order that there is a valid waiver to a warrantless search, the waiver or consent should be given by the person
affected, not just anybody. Example: The landlady could not give a valid consent to the search of a room occupied
by a tenant. Said tenant himself should give the consent in order to be valid. The doctrine in Lopez vs.
Commissioner to the effect that it could be given by any occupant of a hotel room being rented by the respondent is
deemed abandoned

6. STOP & FRISK; SEARCH IN CHECKPOINTS


SUSAN ESQUILLO VS PEOPLE, G.R. No. 182010, August 25, 2010
Stop and Friskvalid exception to a Warrantless search and seizure
If the person who claims that he was illegally arrested did not question the validity of her Warrantless arrest before
her arraignment nor moved to quash the information but raised said issues for the first time on appeal, the same is
already deemed waived.

POSADAS VS CA, 188 SCRA 288


Patrolmans Ungab and Umpar, both members of the INP of the Davao Metrodiscom assigned w/ the Intelligence
Task Force, were conducting a surveillance along Magallanes, St., Davao City. While they were w/in the premises
of the Rizal Memorial Colleges, they spotted petitioner carrying a "buri" bag & they noticed him to be acting
suspiciously. They approached the petitioner and identified themselves as members of the INP. Petitioner
attempted to flee but was stopped by the 2. They then checked the "buri" bag of the petitioner where they found 1
caliber .38 Smith & Wesson revolver, w/ 2 rounds of live ammunition for a .38 cal. gun, a smoke grenade, & 2 live
ammunition for a .22 caliber gun. Petitioner was brought to the police station for further investigation. He was
prosecuted for illegal possession of firearms and ammunitions in the RTC of Davao City wherein after a plea of not
guilty, and trial on the merits, a decision was rendered finding petitioner guilty. The CA affirmed the appealed
decision in toto.
Hence, the petition for review, the main thrust of w/c is that there being no lawful arrest or search and seizure, the
items w/c were confiscated from the possession of the petitioner are inadmissible in evidence against him.
The Sol-Gen argues that under Sec. 12, R 136 of ROC, a person lawfully arrested may be searched for dangerous
weapons or anything (w/c may be) used as proof of a commission of an offense, w/o a SW.
HELD: From Sec. 5, R 113, ROC, it is clear that an arrest w/o a warrant may be effected by a peace officer or
private person, among others, when in his presence the person to be arrested has committed, is actually
committing, or is attempting to commit an offense, or when an offense has in fact, just been committed, & he has
personal knowledge of the facts indicating that the person arrested has committed it.
At the time the peace officers identified themselves and apprehended the petitioner as he attempted to flee, they
did not know that he had committed, or was actually committing, the offense. They just suspected that he was
hiding something in the buri bag. They did not know what its contents were. The said circumstances did not justify
an arrest w/o a warrant.
However, there are many instances where a warrant & seizure can be effected w/o necessarily being preceded by
an arrest, foremost of w/c is the 'stop & search' w/o a SW at military or police checkpoints, the constitutionality of
w/c has been upheld by this Court in Valmonte v. de Villa.
As bet. a warrantless search and seizure (S & S) conducted at military or police checkpoints and the search thereof
in the case at bar, there is no question that, indeed, the latter is more reasonable considering that, unlike in the
former, it was effected on the basis of a probable cause. The probable cause is that when the petitioner acted

suspiciously and attempted to flee w/ the buri bag, there was a probable cause that he was concealing something
illegal in the bag and it was the right and duty of the police officers to inspect the same.
It is too much indeed to require the police officers to search the bag in the possession of the petitioner only after
they shall have obtained a SW for the purpose. Such an exercise may prove to be useless, futile and much too
late.
Waiver cannot be implied from the fact that the person consented or did not object to the search, for it many
happen that he did so only out of respect for the authorities. The waiver must be expressly made.
MANALILI VS CA, October 9, 1997
The policemen saw several suspicion-looking men at dawn and when they approached said persons, they ran but
were caught. The unlicensed firearm confiscated after the policemen search them is admissible.
Narcotics officers were doing surveillance and chanced upon the accused in a cemetery who seemed to be high on
drugs. He tried to resist the police officers and upon inquiry, found that the accused was possessing what
seemed to be crushed marijuana leaves.
HELD: A stop-and-frisk was defined as the vernacular designation of the right of a police officer to stop a citizen on
the street, interrogate him, and pat him for weapons. It has been held as one of the exceptions to the general rule
against searches without warrant.
MALACAT VS CA, 283 SCRA 159
ISSUE: Whether the search made on Malacat is valid, pursuant to the exception of stop and frisk.
HELD: The general rule regarding arrests, searches and seizures is that a warrant is needed in order to validly
effect the same. The Constitutional prohibition against unreasonable arrests, searches and seizures refers to those
effected without a validly issued warrant, subject to certain exceptions. As regards valid warrantless arrests, these
are found in Section 5, Rule 113 of the Rules of Court. A warrantless arrest under the circumstances contemplated
under Section 5(a) has been denominated as one in flagrante delicto, while that under Section 5(b) has been
described as a hot pursuit arrest. Turning to valid warrantless searches, they are limited to the following:
(1) customs searches;
(2) search of moving vehicles;
(3) seizure of evidence in plain view;
(4) consent searches;
(5) a search incidental to a lawful arrest; and
(6) a stop and frisk.
The concepts of a stop-and-frisk and of a search incidental to a lawful arrest must not be confused. These two
types of warrantless searches differ in terms of the requisite quantum of proof before they may be validly effected
and in their allowable scope. In a search incidental to a lawful arrest, as the precedent arrest determines the validity
of the incidental search. Here, there could have been no valid in flagrante delicto or hot pursuit arrest preceding the
search in light of the lack of personal knowledge on the part of Yu, the arresting officer, or an overt physical act, on
the part of Malacat, indicating that a crime had just been committed, was being committed or was going to be
committed. Plainly, the search conducted on Malacat could not have been one incidental to a lawful arrest. On the
other hand, while probable cause is not required to conduct a stop and frisk, it neverthe less holds that mere
suspicion or a hunch will not validate a stop and frisk. A genuine reason must exist, in light of the police officers
experience and surrounding conditions, to warrant the belief that the person detained has weapons concealed
about him. Finally, a stop-and-frisk serves a two-fold interest: (1) the general interest of effective crime prevention
and detection, which underlies the recognition that a police officer may, under appropriate circumstances and in an
appropriate manner, approach a person for purposes of investigating possible criminal behavior even without
probable cause; and (2) the more pressing interest of safety and self-preservation which permit the police officer to
take steps to assure himself that the person with whom he deals is not armed with a deadly weapon that could
unexpectedly and fatally be used against the police officer.
Here, there are at least three (3) reasons why the stop-and-frisk was invalid: First, there is grave doubts as to Yus
claim that Malacat was a member of the group which attempted to bomb Plaza Miranda 2 days earlier. This claim is
neither supported by any police report or record nor corroborated by any other police officer who allegedly chased
that group. Second, there was nothing in Malacats behavior or conduct which could have reasonably elicited even
mere suspicion other than that his eyes were moving very fast an observation which leaves us incredulous
since Yu and his teammates were nowhere near Malacat and it was already 6:30 p.m., thus presumably dusk.
Malacat and his companions were merely standing at the corner and were not creating any commotion or trouble.
Third, there was at all no ground, probable or otherwise, to believe that Malacat was armed with a deadly weapon.
None was visible to Yu, for as he admitted, the alleged grenade was discovered inside the front waistline of
Malacat, and from all indications as to the distance between Yu and Malacat, any telltale bulge, assuming that
Malacat was indeed hiding a grenade, could not have been visible to Yu. What is unequivocal then are blatant
violations of Malacats rights solemnly guaranteed in Sections 2 and 12(1) of Article III of the Constitution.

GUAZON ET AL VS DE VILLA ET AL, GR 80508


This is a petition for Prohibition with preliminary injunction to prohibit military and police officers from conducting
Areal target zonings or saturation drive in Metro Manila particularly in places where they suspect that the
subversives are hiding. The 41 petitioners claim that the saturation drives conducted by the military is in violation of
their human rights because with no specific target house in mind, in the dead of the night or early morning hours,
police and military officers without any search warrant cordon an area of more than one residence and sometimes
the whole barangay. Most of them are in civilian clothes and w/o nameplates or identification cards; that the raiders
rudely rouse residents from their sleep by banging on the walls and windows of their homes, shouting, kicking their
doors open (destroying some) and ordering the residents to come out; the residents are herded like cows at the
point of high powered guns, ordered to strip down to their briefs and examined for tattoo marks; that while
examination of the bodies of the men are being conducted, the other military men conduct search and seizures to
each and every house without civilian witnesses from the neighbors; some victims complained that their money and
other valuables were lost as a result of these illegal operations.
The respondents claim that they have legal authority to conduct saturation drives under Art. VII, Sec. 17 of the
Constitution which provides:
The respondents would want to justify said military operation on the following constitutional provisions:
xxxxxx
The President shall be the Commander-in-Chief of all the armed forces of the Philippines and whenever it
becomes necessary, he may call out such armed forces to prevent or suppress lawless violence, invasion or
rebellion
xxxxxx
The President shall have control of all the executive departments, bureaus and offices. He shall ensure that the
laws are faithfully executed.
xxxxxx
HELD: There can be no question that under ordinary circumstances, the police action of the nature described by
the petitioners would be illegal and blatantly violative of the Bill of Rights. If the military wants to flush out
subversive and criminal elements, the same must be consistent with the constitutional and statutory rights of the
people. However, nowhere in the Constitution can we see a provision which prohibits the Chief Executive from
ordering the military to stop unabated criminality, rising lawlessness and alarming communist activities. However,
all police actions are governed by the limitations of the Bill of Rights. The government cannot adopt the same
reprehensible methods of authoritarian systems both of the right and of the left. This is so because Art. III, Section
3 of the Constitution is very clear as explained in Roan vs. Gonzales, 145 SCRA 687 and Century Fox vs. Court of
Appeals, 164 SCRA 655. Also, it must be pointed out that police actions should not be characterized by methods
that offend ones sense of justice (Rochin vs. California, 342 US 165).
The Court believes it highly probable that some violations were actually committed. But the remedy is not to stop all
police actions, including the essential and legitimate ones. A show of force is sometimes necessary as long as the
rights of people are protected and not violated. However, the remedy of the petitioners is not an original action for
prohibition since not one victim complains and not one violator is properly charged. It is basically for the executive
department and the trial courts. The problem is appropriate for the Commission of Human Rights.
The petition was therefore remanded to the Regional Trial Courts of Manila, Malabon and Pasay City where the
petitioners may present evidence supporting their allegations so that the erring parties may be pinpointed and
prosecuted. In the meantime, the acts violative of human rights alleged by the petitioners as committed during the
police actions are ENJOINED until such time as permanent rules to govern such actions are promulgated.

IN THE MATTER OF THE PETITION FOR HABEAS CORPUS OF ROBERTO VILLANUEVA, GR 81567

IF THE JUDGE FINDS THAT THERE'S PROBABLE CAUSE, MUST HE ISSUE A WARRANT OF ARREST AS A
MATTER OF COURSE?

SAMULDE VS SALVANI, SEPTEMBER 26, 1988


No, because a warrant is issued in order to have jurisdiction of the court over the person of an accused and to
assure the court of his presence whenever his case is called in court. As such, if the court believes that the
presence of the accused could be had even without a warrant of arrest, then he may not issue said warrant. Note:
This case involves a minor offense.
GOZO VS TAC-AN, 300 SCRA 265
If the offense committed is a serious one like that obtaining in this case for murder, the Judge must issue a warrant
of arrest after determining the existence of probable cause

PLACE TO SEARCH INDICATED IN THE WARRANT IS DIFFERENT FROM WHAT THE POLICE HAD IN MIND
WHEN APPLIED FOR WARRANT
PEOPLE VS COURT OF APPEALS, 291 SCRA 400
In applying for a search warrant, the police officers had in their mind the first four (4) separate apartment units at
the rear of ABIGAIL VARIETY STORE in Quezon City to be the subject of their search. The same was not,
however, what the Judge who issued the warrant had in mind, AND WAS NOT WHAT WAS ULTIMATELY
DESCRIBED IN THE SEARCH WARRANT. As such, any evidence obtained from the place searched which is
different from that indicated in the search warrant is inadmissible in evidence for any purpose and in any
proceeding.
This is so because it is neither licit nor fair to allow police officers to search a place different from that stated in the
warrant on the claim that the place actually searchedalthough not that specified in the search warrantis exactly
what they had in view when they applied for the warrant and had demarcated in their supporting evidence. WHAT
IS MATERIAL IN DETERMINING THE VALIDITY OF A SEARCH IS THE PLACE STATED IN THE WARRANT
ITSELF, NOT WHAT THE APPLICANTS HAD IN THEIR THOUGHTS, OR HAD REPRESENTED IN THE
PROOFS THEY SUBMITTED TO THE COURT ISSUING THE WARRANT. As such, it was not just a case of
obvious typographical error, but a clear case of a search of a place different from that clearly and without
ambiguity identified in the search warrant.
NOTE: Very Important: Where a search warrant is issued by one court and the criminal action base don the results
of the search is afterwards commenced in another court, IT IS NOT THE RULE THAT A MOTION TO QUASH THE
WARRANT (or to retrieve the things seized) MAY BE FILED ONLY IN THE ISSUING COURTSUCH A MOTION
MAY BE FILED FOR THE FIRST TIME IN EITHER THE ISSUING COURT OR THAT IN WHICH THE CRIMINAL
PROCEEDING IS PENDING.

SEARCHING QUESTIONS
PRUDENTE VS DAYRIT, GR 82870
Search warrant was issued because of a very reliable tip obtained by the police against Dr. Nemesio Prudente,
President of Polytechnic University of the Philippines and implemented on November 1, 1987, a Sunday and All
Saints Day despite Circular No. 19 of the SC.
This is a petition to annul and set aside the Order of respondent Judge DENYING the motion of the petitioner to
quash Search Warrant No. 87-14 as well as its Order denying the petitioners Motion for Reconsideration.
1. On October 31, 1987, P/Major Alladin Dimagmaliw, Chief of the Intelligence Special Action Division (ISAD) of
the Western Police District (WPD) filed with the Regional Trial Court of Manila, Branch 33, presided by the
respondent Judge, an application for the issuance of a Search Warrant for violation of PD 1866 against the
petitioner;
2. In his application for search warrant, P/Major Dimagmaliw alleged that:
1. That he has been informed and has good and sufficient reasons to believe that NEMESIO
PRUDENTE who may be found at the Polytechnic University of the Philippines x x x has in his
control or possession firearms, explosives, hand grenades and ammunition intended to be used as
the means of committing an offense x x x;
2. That the undersigned has verified the report and found it to be a fact x x x .
In support of said application, P/Lt. Florencio Angeles executed a Deposition of Witness dated October 31,
1987
3. On November 1, 1987, a Sunday and All Saints Day, the search warrant was enforced by some 200 WPD
operatives led by Col. Edgar Dula Torre and Major Maganto;
4. On November 2, 1987, Ricardo Abando, a member of the searching team executed an affidavit alleging that
he found in the drawer of a cabinet inside the wash room of Dr. Prudentes office a bulging brown envelope
with three live fragmentation hand grenades separately with old newspapers;
5. On November 6, 1987, the petitioner moved to quash the search warrant on the grounds that:
a. the complainants lone witness, Lt. Angeles had no personal knowledge of the facts which formed the
basis for the issuance of the search warrant;
b. the examination of said witness was not in the form of searching questions and answers;
c. the search warrant was a general warrant, for the reason that it did not particularly describe the place
to be searched and that it failed to charge one specific offense; and
d. the warrant was issued in violation of Circular No. 19 of the Supreme Court in that the complainant
failed to allege that the issuance of the search warrant on a Saturday was urgent.
6. On March 9, 1986, the respondent judge denied the motion to quash and on April 20, 1988, the same judge
denied petitioners motion for reconsideration. Hence this petition.
ISSUE: Was the Search Warrant issued by the respondent judge valid? Was there probable cause?
HELD:
a. For a valid search warrant to issue, there must be probable cause, which is to be determined by the judge,
after examination under oath or affirmation of the complainant and the witnesses he may produce, and
particularly describing the place to be searched and the persons or things to be seized. The probable cause
must be in connection with one specific offense and the judge must, before issuing the warrant, personally
examine in the form of searching questions and answers, in writing and under oath, the complainant and the
witnesses he may produce, on facts personally known to them and attach to the record their sworn
statements together with any affidavits submitted.
The probable cause for a valid search warrant, has been defined as such facts and circumstances which
would lead a reasonably discreet and prudent man to believe that an offense has been committed, and that
the objects sought in connection with the offense are in the place sought to be searched. (Quintero vs. NBI,
June 23, 1988). This probable cause must be shown to be within the personal knowledge of the complainant
or the witnesses he may produce and not based on mere hearsay. (P. VS. SY JUCO, 64 PHIL. 667;
ALVAREZ VS. CFI, 64 PHIL. 33; US VS. ADDISON, 28 PHIL. 566).

In his affidavit, Major Dimagmaliw declared that he has been informed that Nemesio Prudente has in his
control and possession the firearms and explosivees described therein, and that he has verified the report
and found it to be a fact. On the other hand, Lt. Angeles declared that as a result of continuous surveillance
for several days, they gathered informations from verified sources that the holders of said firearms and
explosives are not licensed t possess them. It is clear from the foregoing that the applicant and his witness
HAD NO PERSONAL KNOWLEDGE OF THE FACTS AND CIRCUMSTANCES which became the basis for
issuing the questioned search warrant, but acquired knowledge thereof only through information from other
sources or persons.
Despite the fact that Major Dimagmaliw stated in his affidavit that he verified the information he had earlier
received and found it to be a fact, YET THERE IS NOTHING IN THE RECORD TO SHOW OR INDICATE
HOW AND WHEN SAID APPLICANT VERIFIED THE EARLIER INFORMATION ACQUIRED BY HIM AS TO
JUSTIFY HIS CONCLUSION. He might have clarified this point if there had been searching questions and
answers, but there were none. In fact, the records yield no questions and answers, whether searching or not,
vis-a-vis the said applicant.
In ALVAREZ VS. CFI, 64 PHIL. 33, it was held that the following test must be complied with in an application
for search warrant or in a supporting deposition based on personal knowledge or notThe true test of sufficiency of a deposition or affidavit to warrant issuance of a search warrant is whether it
was drawn in a manner that perjury could be charged thereon and the affiant be held liable for damage
caused. The oath required must refer to the truth of the facts within the personal knowledge of the applicant
of a search warrant and/or his witnesses, not of the facts merely reported by a person whom one considers to
be reliable.
Tested by the above standards, the allegation of the witness, Lt. Angeles, do not come up to the level of facts
based on his personal knowledge so much so that he cannot be held liable for perjury for such allegations in
causing the issuance of the questioned search warrant.
Besides, respondent judge did not take the deposition of the applicant as required by the Rules of Court. As
held in Roan vs. Gonzales, 145 SCRA 694, mere affidavits of the complainant and his witnesses are thus
insufficient. The examining judge has to take the depositions in writing of the complainant and the witnesses
he may produce and attach them to the record.
b. There was also no searching questions asked by the respondent judge because as shown by the record, his
questions were too brief and short and did not examine the complainant and his witnesses in the form of
searching questions and answers. On the contrary, the questions asked were leading as they called for a
simple yes or no answer. As held in Quintero vs. NBI, June 23, 1988, the questions propounded are not
sufficiently searching to establish probable cause. Asking of leading questions to the deponent in an
application for search warrant and conducting of examination in a general manner would not satisfy the
requirements for the issuance of a valid search warrant.
The Court avails of this decision to reiterate the strict requirements for determination of probable cause in the
valid issuance of a search warrant as enunciated in earlier cases. True, this requirements are stringent but
the purpose is to assure that the constitutional right of the individual against unreasonable search and seizure
shall remain both meaningful and effective.
c.

The rule is, that a description of a place to be searched is sufficient if the officer with the warrant can with
reasonable effort ascertain and identify the place intended (P VS. VELOSO, 48 PHIL. 180). In the case at
bar, the warrant described the place to be searched as the premises of the PUP, more particularly the offices
of the Department of Science and Tactics as well as the Office of the President, Nemesio Prudente.
There is also no violation of the one specific offense requirement considering that the application for a
search warrant explicitly described the offense: illegal possession of firearms and ammunitions under PD
1866.
d. CIRCULAR NO. 19 OF THE SUPREME COURT merely provides for a guideline, departure from which
would not necessarily affect the validity of the search warrant provided the constitutional requirements are
complied with.

WEBB VS DE LEON, 247 SCRA 650

ALVAREZ VS CFI, 64 PHIL 33


When the applicant is basing his knowledge from an informant, the same is not valid
On 3 June 1936, the chief of the secret service of the Anti-Usury Board, of the Department of Justice, presented to
Judge Eduardo Gutierrez David then presiding over the Court of First Instance of Tayabas, an affidavit alleging that
according to reliable information, Narciso Alvarez kept in his house in Infanta, Tayabas, books, documents,
receipts, lists, chits and other papers used by him in connection with his activities as a moneylender, charging
usurious rates of interest in violation of the law. In his oath at the end of the affidavit, the chief of the secret service
stated that his answers to the questions were correct to the best of his knowledge and belief. He did not swear to
the truth of his statements upon his own knowledge of the facts but upon the information received by him from a
reliable person. Upon the affidavit the judge, on said date, issued the warrant which is the subject matter of the
petition, ordering the search of the Alvarezs house at any time of the day or night, the seizure of the books and
documents and the immediate delivery thereof to him to be disposed of in accordance with the law. With said
warrant, several agents of the Anti-Usury Board entered Alvarezs store and residence at 7:00 p.m. of 4 June 1936,
and seized and took possession of the following articles: internal revenue licenses for the years 1933 to 1936, 1
ledger, 2 journals, 2 cashbooks, 9 order books, 4 notebooks, 4 check stubs, 2 memorandums, 3 bankbooks, 2
contracts, 4 stubs, 48 stubs of purchases of copra, 2 inventories, 2 bundles of bills of lading, 1 bundle of credit
receipts, 1 bundle of stubs of purchases of copra, 2 packages of correspondence, 1 receipt book belonging to Luis
Fernandez, 14 bundles of invoices and other papers, many documents and loan contracts with security and
promissory notes, 504 chits, promissory notes and stubs of used checks of the Hongkong & Shanghai Banking
Corporation (HSBC). The search for and seizure of said articles were made with the opposition of Alvarez who
stated his protest below the inventories on the ground that the agents seized even the originals of the documents.
As the articles had not been brought immediately to the judge who issued the search warrant, Alvarez, through his
attorney, filed a motion on 8 June 1936, praying that the agent Emilio L. Siongco, or any other agent, be ordered
immediately to deposit all the seized articles in the office of the clerk of court and that said agent be declared guilty
of contempt for having disobeyed the order of the court. On said date the court issued an order directing Siongco to
deposit all the articles seized within 24 hours from the receipt of notice thereof and giving him a period of 5 days
within which to show cause why he should not be punished for contempt of court. On 10 June, Attorney Arsenio
Rodriguez, representing the Anti-Usury Board, filed a motion praying that the order of the 8th of said month be set
aside and that the Anti-Usury Board be authorized to retain the articles seized for a period of 30 days for the
necessary investigation. On June 25, the court issued an order requiring agent Siongco forthwith to file the search
warrant and the affidavit in the court, together with the proceedings taken by him, and to present an inventory duly
verified by oath of all the articles seized. On July 2, the attorney for the petitioner filed a petition alleging that the
search warrant issued was illegal and that it had not yet been returned to date together with the proceedings taken
in connection therewith, and praying that said warrant be cancelled, that an order be issued directing the return of
all the articles seized to Alvarez, that the agent who seized them be declared guilty of contempt of court, and that
charges be filed against him for abuse of authority. On September 10, the court issued an order holding: that the
search warrant was obtained and issued in accordance with the law, that it had been duly complied with and,
consequently, should not be cancelled, and that agent Siongco did not commit any contempt of court and must,
therefore, be exonerated, and ordering the chief of the Anti-Usury Board in Manila to show cause, if any, within the
unextendible period of 2 days from the date of notice of said order, why all the articles seized appearing in the
inventory should not be returned to Alvarez. The assistant chief of the Anti-Usury Board of the Department of
Justice filed a motion praying, for the reasons stated therein, that the articles seized be ordered retained for the
purpose of conducting an investigation of the violation of the Anti-Usury Law committed by Alvarez. On October 10,
said official again filed another motion alleging that he needed 60 days to examine the documents and papers
seized, which are designated on pages 1 to 4 of the inventory by Nos. 5, 10, 16, 23, 25-27, 30-31 , 34, 36-43 and
45, and praying that he be granted said period of 60 days. In an order of October 16, the court granted him the
period of 60 days to investigate said 19 documents. Alvarez, herein, asks that the search warrant as well as the
order authorizing the agents of the Anti-Usury Board to retain the articles seized, be declared illegal and set aside,
and prays that all the articles in question be returned to him.
ISSUE: Whether the search warrant issued by the court is illegal because it has been based upon the affidavit of
agent Almeda in whose oath he declared that he had no personal knowledge of the facts which were to serve as a
basis for the issuance of the warrant but that he had knowledge thereof through mere information secured from a
person whom he considered reliable, and that it is illegal as it was not supported by other affidavits aside from that
made by the applicant.
HELD: Section 1, paragraph 3, of Article III of the Constitution and Section 97 of General Orders 58 require that
there be not only probable cause before the issuance of a search warrant but that the search warrant must be
based upon an application supported by oath of the applicant and the witnesses he may produce. In its broadest
sense, an oath includes any form of attestation by which a party signifies that he is bound in conscience to perform
an act faithfully and truthfully; and it is sometimes defined as an outward pledge given by the person taking it that
his attestation or promise is made under an immediate sense of his responsibility to God. The oath required must

refer to the truth of the facts within the personal knowledge of the petitioner or his witnesses, because the purpose
thereof is to convince the committing magistrate, not the individual making the affidavit and seeking the issuance of
the warrant, of the existence of probable cause. The true test of sufficiency of an affidavit to warrant issuance of a
search warrant is whether it has been drawn in such a manner that perjury could be charged thereon and affiant be
held liable for damages caused. The affidavit, which served as the exclusive basis of the search warrant, is
insufficient and fatally defective by reason of the manner in which the oath was made, and therefore, the search
warrant and the subsequent seizure of the books, documents and other papers are illegal. Further, it is the practice
in this jurisdiction to attach the affidavit of at least the applicant or complainant to the application. It is admitted that
the judge who issued the search warrant in this case, relied exclusively upon the affidavit made by agent Almeda
and that he did not require nor take the deposition of any other witness. Neither the Constitution nor General
Orders 58 provides that it is of imperative necessity to take the depositions of the witnesses to be presented by the
applicant or complainant in addition to the affidavit of the latter. The purpose of both in requiring the presentation of
depositions is nothing more than to satisfy the committing magistrate of the existence of probable cause. Therefore,
if the affidavit of the applicant or complainant is sufficient, the judge may dispense with that of other witnesses.
Inasmuch as the affidavit of the agent was insufficient because his knowledge of the facts was not personal but
merely hearsay, it is the duty of the judge to require the affidavit of one or more witnesses for the purpose of
determining the existence of probable cause to warrant the issuance of the search warrant. When the affidavit of
the applicant or complainant contains sufficient facts within his personal and direct knowledge, it is sufficient if the
judge is satisfied that there exists probable cause; when the applicants knowledge of the facts is mere hearsay, the
affidavit of one or more witnesses having a personal knowledge of the facts is necessary. Thus the warrant issued
is likewise illegal because it was based only on the affidavit of the agent who had no personal knowledge of the
facts.

LUNA VS PLAZA, 26 SCRA 313

DA MULATA VS IRIZARI, 62 SCRA 210

MARINAS VS SIOCHI, 104 SCRA 423

ROAN VS GONZALES, 145 SCRA 687

MATA VS BAYONA, 128 SCRA 388


The deposition of the complainant and the witnesses shall be attached to the search warrant as well as in the
record of the case, without which, the search for masiao tickets is not valid.

CORRO VS LISING, 137 SCRA 541

NOLASCO VS PANO 147 SCRA 509

BURGOS VS CHIEF OF STAFF, 133 SCRA 800


Mere conclusions of law by the applicant that there are subversive documents at the offices of the newspaper WE
FORUM during martial law do not justify a finding of probable cause.

PEOPLE VS BURGOS, September 14, 1986

PEOPLE VS AMINNUDIN, July 6, 1988

PONSICA VS IGNALAGA, July 31, 1987

If the statements of the complainant and the witnesses of an applicant for a search warrant are mere generalities,
mere conclusions of law and not positive statements of particular facts, the search warrant is not valid.

ABERCA VS VER, April 15, 1988


The superiors, including Capt. Panfilo Lacson, are liable civilly for the illegal search conducted by their
subordinates.
PANGANIBAN VS CESAR, 159 SCRA 599

WARRANTLESS SEARCHES AND SEIZURES WHEN VALID OR NOT


VALMONTE VS DE VILLA, GR 83988
1. On January 20, 1987, the National Capital Region District Command (NCRDC) was activated with the
mission of conducting security operations within its area of responsibility for the purpose of maintaining
peace and order. As part of its duty to maintain peace and order, the NCRDC installed checkpoints in
various parts of Valenzuela, Metro Manila.
Petitioners claim that because of these checkpoints, the residents of Valenzuela, MM are worried of being
harassed and of their safety being placed at the arbitrary, capricious and whimsical disposition of the
military authorities manning the checkpoints considering that their cars and vehicles are being subjected to
regular searches and check-ups, especially at night or dawn, without the benefit of a search warrant and/or
court order.
2. On July 9, 1988 at dawn, the apprehensions of the residents of Valenzuela increased because Benjamin
Parpon, the supply officer of the Municipality of Valenzuela was gunned down in cold blood by the military
men manning the checkpoints for ignoring or refusing to submit himself to the checkpoint and for continuing
to speed off inspite of several warning shots fired in the air.
ISSUE: Whether or not the existence of said checkpoints as well as the periodic searches and seizures made by
the military authorities without search warrant valid?
HELD: Petitioners concern for their safety and apprehension at being harassed by the military manning the
checkpoints are not sufficient grounds to declare the checkpoints as per se illegal.
Not all searches and seizures are prohibited. Those which are reasonable are not forbidden. A reasonable search
is not to be determined by any fixed formula but is to be resolved according to the facts of each case.
Where, for example, the officer merely draws aside the curtain of a vacant vehicle which is parked on a public fair
grounds (People vs. Case, 190 MW 289), or simply looks into a vehicle (State vs. Gaina, 97 SE 62), or flashes a
light therein (Rowland vs. Commonwealth, 259 SW 33), these do not constitute unreasonable search.
The setting up of checkpoints in Valenzuela, Metro Manila may be considered as security measure to effectively
maintain peace and order and to thwart plots to destabilize the government. In this connection, the Court may take
judicial notice of the shift to urban centers and their suburbs of the insurgency movement, so clearly reflected in the
increased killings in cities of police and military men by NPAs sparrow units, not to mention the abundance of
unlicensed firearms.
BETWEEN THE INHERENT RIGHT OF THE STATE TO PROTECT ITS EXISTENCE AND PROMOTE PUBLIC
WELFARE AND AN INDIVIDUALS RIGHT AGAINST A WARRANTLESS SEARCH WHICH IS HOWEVER
REASONABLY CONDUCTED, THE FORMER SHALL PREVAIL.
True, the manning of these checkpoints by the military is susceptible of abuse by the men in uniform, in the same
manner that all governmental power is susceptible to abuse. BUT , AT THE COST OF OCCASIONAL
INCONVENIENCE, DISCOMFORT AND EVEN IRRITATION TO THE CITIZEN, THE CHECKPOINTS DURING
THESE ABNORMAL TIMES ARE PART OF THE PRICE WE PAY FOR AN ORDERLY SOCIETY AND PEACEFUL
COMMUNITY.
Finally, it must be emphasized that on July 17, 1988, the military checkpoints in Metro Manila were temporarily
lifted and a review and refinement of the rules in the conduct of the police and military manning the checkpoints
upon order of the NCRDC Chief.

RESOLUTION ON THE MOTION FOR RECONSIDERATION, JUNE 15, 1990


The Supreme Court in its Resolution of the Motion for Reconsideration dated 15 June, 1990, held that military
and police checkpoints are not illegal as these measures to protect the government and safeguards the lives of the
people. The checkpoints are legal as where the survival of the organized government is on the balance, or where
the lives and safety of the people are in grave peril. However, the Supreme Court held further that the military
officers manning the checkpoints may conduct VISUAL SEARCH ONLY, NOT BODILY SEARCH.

RIZAL ALIH VS GENERAL CASTRO, June 23,1987

PEOPLE VS CENDANA, October 17, 1990

PEOPLE VS CASTILLER, August 6, 1990

PEOPLE VS OLAES, July 30, 1990

PAPA VS MAGO, 22 SCRA 857

ROLDAN VS ARCA, 65 SCRA 336

PEOPLE VS CFI, 101 SCRA 86

PACIS VS PAMARAN, 56 SCRA 16

LOPEZ VS COMMISSIONER, 65 SCRA 336

PEOPLE VS CRUZ, 165 SCRA 135

NOLASCO VS PANO, 147 SCRA 509 & 139 SCRA 152

PEOPLE VS CLAUDIO, 160 SCRA 646


There is a valid warrantless search if a NARCOM officer arrests the person who owns a bag which contains
marijuana which he found out when he smelled the same. Here , there is a probable cause since he was personal
knowledge due to his expertise on drugs.

PEOPLE VS DEL ROSARIO, July 10, 1994.

MAY A NON-JUDICIAL OFFICER ISSUE A WARRANT OF ARREST?


HARVEY VS DEFENSOR-SANTIAGO, June 26,1988

MORENO VS VIVO, 20 SCRA 562

LIM VS DE LEON, 66 SCRA 299

SALAZAR VS ACHACOSO, GR 81510,

PRESIDENTIAL ANTI_DOLLAR SALTING TASK FORCE VS COURT OF APPEALS, March 16, 1989

PROPERTIES SUBJECT TO SEIZURE


SEC. 2, RULE 126, 1985 RULES ON CRIMINAL PROCEDURE, AS AMENDED

ESPANO VS COURT OF APPEALS, 288 SCRA 558

WARRANTLESS SEARCHES AND ARRESTS


PEOPLE VS BATI, August 27, 1990

MANUEL ET AL VS VELASCO, GR No. 84666

GARCIA-PADILLA VS ENRILE,121 SCRA 47 & 137 SCRA 647

PEOPLE VS MASPIL JR., August 20, 1990 (Compare with P. vs. Aminnudin, July 6, 1988, supra)

POSADAS VS COURT OF APPEALS, Aug. 2, 1990

PEOPLE VS ORTIZ, December 3, 1990

GO VS COURT OF APPEALS, February 11, 1992

PEOPLE VS MATI, January 18, 1991

MORALES VS ENRILE, 121 SCRA 538

PEOPLE VS BURGOS, 144 SCRA 1

PEOPLE VS DE LA CRUZ, 184 SCRA 416

GATCHALIAN VS BOARD, May 31, 1991

PEOPLE VS SUCRO, March 18, 1991


Pat. Fulgencio went to Arlie Regalados house at C. Quimpo to monitor activities of Edison SUCRO (accused).
Sucro was reported to be selling marijuana at a chapel 2 meters away from Regalados house. Sucro was
monitored to have talked and exchanged things three times. These activities are reported through radio to P/Lt.
Seraspi. A third buyer was transacting with appellant and was reported and later identified as Ronnie Macabante.

From that moment, P/Lt.Seraspi proceeded to the area. While the police officers were at the Youth Hostel in
Maagama St. Fulgencio told Lt. Seraspi to intercept. Macabante was intercepted at Mabini and Maagama crossing
in front of Aklan Medical center. Macabante saw the police and threw a tea bag of marijuana on the ground.
Macabante admitted buying the marijuana from Sucro in front of the chapel.
The police team intercepted and arrested SUCRO at the corner of C. Quimpo and Veterans. Recovered were 19
sticks and 4 teabags of marijuana from a cart inside the chapel and another teabag from Macabante.
ISSUES:
(1) Whether or Not arrest without warrant is lawful.
(2) Whether or Not evidence from such arrest is admissible.
HELD: Search and seizures supported by a valid warrant of arrest is not an absolute rule. Rule 126, Sec 12 of
Rules of Criminal Procedure provides that a person lawfully arrested may be searched for dangerous weapons or
anything, which may be used as proff of the commission of an offense, without a search warrant.(People v.
Castiller) The failure of the police officers to secure a warrant stems from the fact that their knowledge required
from the surveillance was insufficient to fulfill requirements for its issuance. However, warantless search and
seizures are legal as long as PROBABLE CAUSE existed. The police officers have personal knowledge of the
actual commission of the crime from the surveillance of the activities of the accused. As police officers were the
ones conducting the surveillance, it is presumed that they are regularly in performance of their duties.

PEOPLE VS SOLAYAO, 262 SCRA 255

PEOPLE VS CUISON, 256 SCRA 325

PEOPLE VS DAMASO, 212 SCRA 547

POSADAS VS CA, 258 SCRA 188

PEOPLE VS JUATAN, 260 SCRA 532 (Buy-bust operation)

SECTION 6, RULE 113, 1985 RULES ON CRIMINAL PROCEDURE, AS AMENDED

EFFECT POSTING BAIL OR ENTERING A PLEA DURING THE ARRAIGNMENT, IF THE ARREST WAS
ILLEGAL. (THE ALLEGED ILLEGALITY OF THE ARREST IS DEEMED WAIVED UPON POSTING OF THE BOND
BY THE ACCUSED)
SY VS PEOPLE, GR 182178

PEOPLE VS GALVEZ, 355 SCRA 246


The policeman arrested the accused-appellant on the basis solely of what Reynaldo Castro had told him and not
because he saw the accused-appellant commit the crime charged against him. Indeed, the prosecution admitted
that there was no warrant of arrest issued against accused-appellant when the latter was taken into custody.
Considering that the accused-appellant was not committing a crime at the time he was arrested nor did the
arresting officer have any personal knowledge of facts indicating that accused-appellant committed a crime, his
arrest without a warrant cannot be justified.
However, by entering a plea of not guilty during the arraignment, the accused-appellant waived his right to raise the
issue of illegality of his arrest. IT IS NOW SETTLED THAT OBJECTION TO A WARRANT OF ARREST OR THE
PROCEDURE BY WHICH A COURT ACQUIRES JURISDICTION OVER THE PERSON OF AN ACCUSED MUST
BE MADE BEFORE HE ENTERS HIS PLEA, OTHERWISE, THE OBJECTION IS DEEMED WAIVED. THE FACT
THAT THE ARREST WAS ILLEGAL DOES NOT RENDER THE SUBSEQUENT PROCEEDINGS VOID AND
DEPRIVE THE STATE OF ITS RIGHT TO CONVICT THE GUILTY WHEN ALL THE FACTS POINT TO THE
CULPABILITY OF THE ACCUSED.

CALLANTA VS VILLANUEVA, 77 SCRA 377

PEOPLE VS NAZARENO, 260 SCRA 256

FILOTEO VS SANDIGANBAYAN, 263 SCRA 222

PEOPLE VS LAPURA, 255 SCRA 85

PEOPLE VS SILAN, 254 SCRA 491

PENALTY FOR ILLEGAL ARREST


PALON VS NAPOLCOM, May 28, 1989

JUDICIAL PRONOUNCEMENTS ON ILLEGALLY SEIZED EVIDENCE, 106 SCRA 336

THE EXCLUSIONARY RULE, 155 SCRA 494

WHAT IS THE STATUS OF A DOCUMENT OBTAINED THROUGH SUBPOENA?


DIANALAN VS PROSECUTORS OFFICE OF THE TANODBAYAN, Nov. 27, 1990

SEARCH WARRANT FOR PIRATED VIDEO TAPES


CENTURY FOX VS COURT OF APPEALS, 164 SCRA 655
The master copy of the allegedly pirated tape should be presented before the judge in order to convince him of the
existence of probable cause.
COLUMBIA PICTURES VS COURT OF APPEALS, 261 SCRA 144

Das könnte Ihnen auch gefallen